Download as pdf or txt
Download as pdf or txt
You are on page 1of 93

REVIEW QUESTIONS PAEDIATRIC

MULTIPLE CHOICE QUESTIONS


1. Which of the following statement is correct regarding HIV and infant feeding?
A. Out of 20 breastfeeding women 5 are likely to be infected
B. In exclusive breastfeeding the mother can give some water
C. Mixed feeding is the best option is mother is working
D. One of the feeding option is exclusive breastfeeding for 6 months
E. Donor milk is recommended in this case

2. A two (2) months old baby was brought to you because of unable to breastfeed for the past
hour. On examination the child is having trouble breathing and appears to be very tired with
bluish discolouration of the tongue. How much oxygen will this child requires as part of the
management?
A. 0.1 to 0.5 litre/minute
B. 0.5 to 1 litre/minute
C. 1 to 1.5 litre/minute
D. 1.5 to 2 litre/minute
E. 2 to 2.5 litre/minute

3. A neonate born with APGAR score of 6 and 9 at 1 and 5 minutes respectively develop
petecheae over the trunk in the first day of life. What is the most likely diagnosis?
A. Hypoxia
B. Hypoglycemia
C. TORCH syndrome
D. Haemorrhagic disease of newborn
E. Congenital haemophilia

4. A 6 years old child with acute onset of fever T=(38.50C) developed febrile seizures which was
treated with anticonvulsants. What would be given to this child to prevent recurrence of
seizure?
A. Paracetamol 400mgs + Diazepam daily
B. Paracetamol 400mgs + Phenobarbitone daily
C. Paracetamol 400mgs hourly
D. I/V Diazepam infusion over 12 hours
E. Oral Diazepam 6 hourly for 24 hours

5. When palpating the liver during abdominal examination


A. It normally palpable in 10% infants
B. It is palpable below the left lateral abdomen subcostal margin
C. Its enlargement can be obtained using scratch test in patient with ascites
D. It is firm and irregular in heart failure
E. It is considered enlarged when span is more than 10-12cm in children

CMT BANK OF QUESTIONS Page 1


REVIEW QUESTIONS PAEDIATRIC

6. Which among the following is a sign of severe dehydration?


A. Weight loss 5-10%
B. Deep respiration
C. Capillary refill > 2 seconds
D. Reduced tear production
E. Thread/impalpable pulse

7. Juma is a known patient of sickle cell disease: he is brought to the hospital with severe anaemia
and transfused one unit of blood. How long will it take for sickling test to be positive?
A. 1 months
B. 2 months
C. 3 months
D. 4 months
E. 6 months

8. A 3 hours old baby was admitted at neonatal care unity with history of seizure, fast breathing
and absent grasp reflex since birth. Upon examination, the child was lethargic, cyanotic,
hypotonic, afebrile, and not jaundiced. Vital signs reveal the child has respiratory rate of 69
breaths/minute. What is the most likely diagnosis?
A. Birth asphyxia
B. Septicemia
C. Neonatal jaundice
D. Delayed milestone
E. Cerebral palsy

9. The most common cause of meningitis in children is: -


A. Malignancy
B. Bacterial
C. Fungal
D. Viral
E. Autoimmune

10. You are a clinician at Namanyere health center where CTC services are provided and your
facility can test HIV antibody test. What is the cut off age at which this test is reliable:
A. 24 months
B. 18 months
C. 36 months
D. 12 months
E. 48 months

11. Failure to grow associated with emaciation and fair appetite which may be defined in term of
weight for length as < 70% or < -3SD is:

CMT BANK OF QUESTIONS Page 2


REVIEW QUESTIONS PAEDIATRIC

A. Metabolic syndrome
B. Marasmus
C. Marasmic Kwashiorkor
D. Kwashiorkor
E. Stunted

12. A four (4) months old body presents with history of fever, inability to breastfeed and vomiting.
On examination he has bulging anterior fontanelle. What is the most likely diagnosis?
A. Severe malaria
B. Meningitis
C. Typhoid fever
D. Urinary tract infection
E. Hydrocephalus

13. Which among the following is a major JONES criterion in diagnosis of acute rheumatic fever?
A. Fever
B. Polyarthritis
C. Arthralgia
D. Presence of C-reactive protein
E. Leukocytosis

14. The most common cause of diarrhea in small children is


A. Vibrio cholera
B. Salmonella
C. Rota virus
D. E.histolytica
E. Shigella

15. Which among the following drug is used as prophylaxis for primary pneumocystis pneumonia,
where cotrimoxazole is contraindicated:
A. Pyridoxine
B. Rifampicin
C. Ethambutal
D. Dapsone
E. Pyrazinamide

16. Which among the following is the earliest sign of nephrotic syndrome:
A. Periorbital edema
B. Scrotal edema
C. Ankle edema
D. Ascites
E. Anasacra

CMT BANK OF QUESTIONS Page 3


REVIEW QUESTIONS PAEDIATRIC

17. The 3 years old boy presents with history of sudden onset of cough and difficult in breathing.
His mother reported that his son was playing with his sister before the symptoms started. On
examination respiratory rate was 55 breaths per minutes and stridor heard on auscultation.
What is the most likely diagnosis:
A. Asthma
B. Pneumonia
C. Foreign body aspiration
D. Bronchiolitis
E. Bronchitis

18. A four days old neonate was brought to the hospital by her mother with history of fever and
yellowish discolouration of the eye, she is the second born and the mother is rhesus negative.
On examination the baby was pale and jaundiced. What is the most likely diagnosis?
A. Congenital red cell aplasia
B. Iron deficiency anaemia
C. Haemorrhagic disease of the newborn
D. Haemolytic disease of the newborn
E. Megaloblastic anaemia

19. Faruku is a 5-year old boy; he presents to you with history of sudden onset of cough and
difficulty in breathing and he prefers half sitting position. On examination he was restless,
cyanotic with audible wheezing sounds. What is the most likely diagnosis?
A. Very severe pneumonia
B. Bronchial asthma
C. Bronchiolitis
D. Severe pneumonia
E. Bronchitis

20. Mwanaidi is 3 years old girl; she was brought to you with history of red discolouration of the
urine. 3 weeks ago she was diagnosed to have group A β haemolytic streptococcal infection,
from which she recovered without treatment. On examination she had lower limb edema and
urine analysis revealed excess protein in the urine. What is the most likely diagnosis:
A. Nephrotic syndrome
B. Sub-acute infective endocarditis
C. Rheumatic fever
D. Acute glomerulonephritis
E. Congenital nephrosis

21. One of the following is the fetal causes of birth asphyxia:


A. Pre-eclampsia
B. Prolonged labour
C. Obstructed labour
D. Prematurity
E. Antepartum haemorrhage

CMT BANK OF QUESTIONS Page 4


REVIEW QUESTIONS PAEDIATRIC

22. Rajabu is a 4 years old boy brought to the hospital with the history of passing watery stool for
three days, in the last 24 hours he had 6 motions. On examination Rajabu is not interested with
what is happening, he has sunken eyes and skin pinch goes back very slowly. What is the most
likely diagnosis?
A. Diarrhoea with no dehydration
B. Diarrhea with severe dehydration
C. Some dehydration
D. Only diarrhea
E. Diarrhea with some dehydration

23. Which among the following is a feature of severe dehydration:


A. Skin pinch goes back slowly
B. The child drinks eagerly
C. Unable to drink or drink poorly
D. Convulsion
E. Restless or irritable

24. A 12 months old baby who weighs 9kg was brought to the OPD with history of high grade
fever and convulsion, while taking history the child convulsed again. What will be the correct
dose of diazepam to this child:
A. 0.3ml rectally
B. 0.5ml rectally
C. 1.0ml rectally
D. 1.25ml rectally
E. 1.5ml rectally

25. A 5 years old boy was brought to hospital by his mother with history of poison ingestion. On
examination the pupils were dilated. What will be the type of poison ingested by this child?
A. Organophosphate poisoning (Note : Constriction of pupil)
B. Phenobarbitone overdose
C. Diazepam overdose
D. Alcohol
E. Aspirin overdose

26. A 13-month old child was brought to a health center with history of excessive crying. She was
vomited three times since morning and passed loose blood stained stool and also draws his
knees up to chest while crying. On examination palpable sausage shaped mass was found at
the umbilical area. What is the most likely diagnosis?
A. Acute volvulus
B. Intestinal obstruction
C. Intussusception
D. Pyloric stenosis
E. Umbilical hernia

CMT BANK OF QUESTIONS Page 5


REVIEW QUESTIONS PAEDIATRIC

27. According to current immunization schedule, PCV 13 is given on:


A. Right shoulder
B. Left shoulder
C. Right thigh
D. Left thigh
E. Drops by mouth.

28. During assessment of Anna’s baby soon after delivery you find out that she is pink, her heart
rate was 90 beat/minutes, regular respiratory effort, some flexion of extremities and weak
response to stimuli. Determine the APGAR score of Anna’s baby:
A. 5
B. 6
C. 7
D. 8
E. 10

29. George is a 5 years old with history of fever and neck stiffness. On examination Brudzinski
and Kerning sign were positive. You decide to do lumber puncture and find out cloudy
cerebral spinal fluid. Identify the most likely cause of the above illness:
A. Viral
B. Fungal
C. Bacterial
D. Protozoa
E. Helminths

30. Mary is 3 years old with 12kgs body weight. She was brought to dispensary by her mother
due to passage of loose watery stool. During physical examination you find out Mary is
lethargic, unable to drink and skin pinch goes back after two seconds. The fluid therapy of
Mary in the next 2½ hours is:
A. 840mls of Ringers lactate
B. 840mls of Normal saline
C. 360mls of Ringers lactate
D. 1200mls of Ringers lactate
E. 1200mls of Normal saline

31. Leila presents to the clinic with her mother , during developmental milestone assessment you
find out that she is able to sit without support rolls from supine to prone, transfer objects from
one hand to another and say “Da” , “Ba”. What is the age to Leila with regard to above
development milestone:
A. 6 weeks
B. 4 months
C. 7 months
D. 10 months
E. 13 months

CMT BANK OF QUESTIONS Page 6


REVIEW QUESTIONS PAEDIATRIC

32. The common causative organism for diarrhea diseases in children is;
A. Fungi
B. Viruses
C. Parasite
D. Bacteria
E. Protozoa

33. Microcytic normochromic anaemia is likely to occur in a child with


A. Sick cell disease
B. Burkitt’s tumor who is on cytotoxic drug therapy
C. Malabsorption syndrome
D. Hookworm infestation
E. Frequent malaria attack

34. In normal milestone development a child of 6 months is able to perform the following fine
motor activities:
A. Begins to babble
B. Coos
C. Ulna grasp
D. Pull at clothes
E. Finger thumb grasp

35. A ten-year-old boy who finished tuberculosis treatment 4 months ago presents at your clinic
with all features of tuberculosis. In which treatment category for TB will he fall
A. Category I
B. Category II
C. Category III
D. Category IV
E. Category V

36. In performing cardiovascular system examination during auscultation you hear a 3rd sound
(Gallop rhythm). This sound is an indication of the presence of the following condition:
A. Acute bronchial asthma
B. Broncho-pneumonia
C. Congestive heart failure
D. Congenital abnormalities
E. Food poisoning

37. Recta prolapse is associated with one of the following heavy worm infestation
A. Schistosomiasis
B. Trichiuriasis
C. Ascariasis
D. Taeniasis
E. Enterobiasis

CMT BANK OF QUESTIONS Page 7


REVIEW QUESTIONS PAEDIATRIC

38. A 3 years old girl, has been experiencing repeated attacks of cough since birth. She does not
tolerate exercises, and with mild exertion she squarts. On examination have features of failure
to thrive, finger clubbing and central cyanosis. Mostly likely the child has one of the following
conditions
A. Acute endocarditis
B. Rheumatic heart diseases
C. Fallot’s tetralogy
D. Atrial septal stenosis
E. Reversed right to left shunt

39. Happy is seven (7) day post-delivery. Her mother brings her to health center due to yellowish
discolouration of eyes that started 4 days ago, it progressed up to the trunk and extremities, 2
days later happy developed convulsion, high pitched cry and inability to breast feed. On
physical examination she is hypotonic and afebrile. Identify the most likely diagnosis
A. Congenial malaria
B. Kernicterus
C. Neonatal sepsis
D. Pathological jaundice
E. Meningitis

40. John is one (1) year old boy. He sustained hot splash while playing in the kitchen. He sustained
burns on half of his right thigh and half of his right leg. What was the total surface area burnt?
A. 3%
B. 4%
C. 5%
D. 6%
E. 7%

41. During rapid screening of sick children at OPD, you find a child who is lethargic, and has
palmar pallor and respiratory distress. This child has
A. Emergency signs
B. Priority signs
C. Non urgency sign
D. To stay at the end of queen
E. No need of prompt assessment

42. Concerning the child with burn injury. One of the following is not the indication for referral
A. 6% full thickness burn
B. 7% full thickness burn
C. 8% partial thickness burn
D. 12% partial thickness burn
E. Involvement of face, hands and feet

CMT BANK OF QUESTIONS Page 8


REVIEW QUESTIONS PAEDIATRIC

43. The following may be applied in management of child who ingested petroleum products
EXCEPT;
A. Support breathing
B. Establish I.V. Saline
C. Specific antidote
D. Gastric lavage
E. Activated charcoal

44. A stridor is characterized by: -


A. Retraction which results in dilatation of nostrils with each breath
B. Musical sounds on expiration due to generalized spasms of bronchial tree
C. A bubbling snoring noise arising from the back of the throat and pharynx
D. A harsh crowing noise due to obstruction or collapse of larynx or trachea.
E. Labored breathing with intercostal spaces indrawing.

45. You have admitted a four years old child with severe pneumonia, later you are called to review
the child and find that has developed severe dypnoea and has an enlarged tender liver. Most
likely the child has developed one of the following conditions:
A. Infective hepatitis
B. Cardiac failure
C. Severe empyema thoracis
D. Pulmonary collapse
E. Liver abscess

46. A 4 years old girl is brought at the clinic with history of vomiting and diarrhea for 3 days. On
examination have sunken eyes, febrile and irritable. When offered some fluid, she drinks
poorly. How will you classify this child?
A. Severe dehydration
B. Some dehydration
C. No dehydration
D. Moderate dehydration
E. Persistent dehydration

47. A six months old child is brought to the clinic with a history of vomiting and passing watery
diarrhea for 3 days. On examination she was very irritable, drinks eagerly and has sunken
eyes. Her body weight is 5kg. How much fluid will you give to this child?
A. 125ml of ORS in every hour for 4 hours
B. 150ml of ORS in every hour for 4 hours
C. 150ml of Ringer’s lactate in the first 1 hour
D. 350ml of Ringer’s lactate in the first 1 hour
E. 375ml of ORS over 4 hours (= 75mls/kg(s) = 375 ml)

CMT BANK OF QUESTIONS Page 9


REVIEW QUESTIONS PAEDIATRIC

48. A 6 years old child has been on antiretroviral drugs for one week now. His mother brings him
to the hospital because she noted a bollous eruption that desquamates. She rash involved soft
skin and mucous membrane. You think this may be a side effect of which drug?
A. Efavirenz
B. Didanozine
C. Lamivudine
D. Niverapine
E. Zidovudine

49. A 5 years old boy presents at OPD with fever, tiredness and pain in the limbs for seven days.
Mother reports of insidious onset. The symptoms have been treated with anti-malaria without
improvement. On examination, the child looks weak, T38.40C, has palmar pallor, tenderness
over the sternum and tibia, enlarged and auxillary lymphnode. From this scenario, this child
may be suffering from:
A. Acute rheumatic fever
B. Juvenile rheumatoid arthritis
C. Acute cell anaemia
D. Sickle cell anaemia
E. Human immunodeficiency virus infection

50. Which type of tuberculosis infection are corticosteroids used as adjuvant therapy
A. Tuberculosis of adrenal gland
B. Bilateral tuberculosis pleural effusion
C. Tuberculosis of spine
D. Milliary tuberculosis
E. Tuberculosis osteomyelitis

51. In performing lumbar puncture, you observe that opening pressure is high and the CSF is
cloudy. What is the most likely diagnosis?
A. TB meningitis
B. Bacterial meningitis
C. Viral meningitis
D. Cryptococcal meningitis
E. Protozoan

52. In performing respiratory system examination, you find that there is decrease in air entry on
the left side of the chest during auscultation. One of the following is the possible cause of the
findings
A. Thing chest cage
B. Lung cavity
C. Lung consolidation
D. Acute bronchial asthma
E. Ribs osteoporosis

CMT BANK OF QUESTIONS Page 10


REVIEW QUESTIONS PAEDIATRIC

53. The management of severe asthmatic attack in children is


A. Subcutaneous adrenaline
B. Intravenous hydrocortisone
C. Oral salbutamol
D. Humidified Oxygen
E. Oral ephedrine

54. Cyanosis in Tetralogy of fallot is caused by:


A. Overriding or aorta
B. Right ventricular hypertrophy
C. Pulmonary stenosis
D. Ventricular septal defect of right – to – left shunt
E. Ventricular septal defect of left – to – right shunt

55. A feature of enlarged spleen that differentiate it from other abdominal masses is that
A. It moves up and down with respiratory
B. It enlarges downwards towards the left iliac fossa
C. It has notch on its medial border
D. It is dull on percussion
E. It is tympanic on percussion

56. In a child with sickle cell anaemia, acute chest syndrome is due to:
A. Vaso-occlusion of cardiac vessels
B. Increased haemolysis of blood in pulmonary vessels
C. Vaso-occlusion of pulmonary vessels
D. Acute inflammatory response in bronchioles
E. Secondary bacterial infection of the lungs

57. Baby Amina, 2 years old, has been on treatment for pneumonia since two days ago. However,
she does not seen to be improving. There is increasing difficulty in breathing, rapid heart rate
and fast breathing. Abdominal examination reveals a soft enlarged liver. Baby Amina is
showing the signs of:
A. Pleural effusion
B. Pulmonary abscess
C. Diaphragmatic abscess
D. Heart failure
E. Pneumothorax

58. In nephrotic syndrome, the type of protein that is lost in greater quantity in urine is
A. Albumin
B. Alphafoetal protein
C. Immunoglobulin
D. Globulin
E. Glucoprotein

CMT BANK OF QUESTIONS Page 11


REVIEW QUESTIONS PAEDIATRIC

59. Amina 3 years old is suffering from cough. The child has been on treatment for 10 days now,
but no improvement. Initially the child was given Cotrimoxazole 240mg 12hourly for 5 days
without improvement. The treatment was then changed to X-pen 1.0MU 6 hourly for 24 hours,
followed by Amoxicillin 250mg 8 hourly for 3 days. However, the condition is detonating.
There is paroxysmal cough with an aspiratory whoop:
The above findings, Amina is suffering from:
A. Laryngotracheobronchitis
B. Severe pneumonia
C. Pulmonary tuberculosis
D. Worm infestation
E. Whooping cough

60. A 3 years old child weighing 12.0kg is brought to the Health Center with a history of fever
and recurrent swelling of the face during morning period. The swelling subsides as the sun
rises. The child passes scanty dark urine. The child is suffering from:
A. Acute glomerulonephritis
B. Nephrotic syndrome
C. Kwashiorkor
D. Marasmus kwashiorkor
E. Urinary tract infection

61. Contraindication of BCG vaccination is


A. All children who are HIV exposure
B. All children who are HIV infected
C. Children with severe malnutrition
D. Children with symptomatic HIV disease
E. Children with previous BCG vaccination without scar

62. During assessiment of sick children at outpatient clinic you find a child whose mother tells
you she has swallowed JIK detergent. How will you triage this child?
A. Emergency
B. Priority
C. Non-urgent
D. Referral case
E. Queue

63. A baby is brought to the clinic with main complain of fever for four days. The mother reports
a history of blood transfusion 3 days ago. The history of blood transfusion will be documented
in which part of your history
A. Main complaints
B. History of presenting illness
C. Past medical history
D. Paediatric history
E. Drug history

CMT BANK OF QUESTIONS Page 12


REVIEW QUESTIONS PAEDIATRIC

64. From which period in weeks is contrimoxazole prophylaxis for pneumocyst jirovecii
pneumonia an HIV exposed child is initiated
A. First
B. Second
C. Third
D. Fourth
E. Sixth

65. A ten year-old who finished anti-tuberculosis treatment 4 months ago presents at your clinic
with all features of tuberculosis. In which treatment category for TB will be fall?
A. 1
B. II
C. III
D. IV
E. V

66. How many weeks recommended for oral iron therapy in anaemia?
A. 4
B. 6
C. 8
D. 10
E. 12

67. A 6 months old child presents at your clinic with central cyanosis and chest indrawing. How
much oxygen in litre(s)/minute will you give?
A. 1
B. 2
C. 3
D. 4
E. 5

68. Among of the following is not the feature of nephrotic syndrome


A. Oedema
B. Proteinuria
C. Hypertension
D. Hyperlipidemia
E. Hypoalbuminaemia

69. Which one of the following is the first line treatment of simple urinary tract infection?
A. Ampicillin
B. Cephalexin
C. Amoxicillin
D. Gentamycin
E. Cotrimoxazole

CMT BANK OF QUESTIONS Page 13


REVIEW QUESTIONS PAEDIATRIC

70. Which fluid will you give to a child in shock with signs of severe malnutrition?
A. Intravenous Dextrose 5%
B. Intravenous Ringer’s lactate
C. Intravenous dextrose saline
D. Oral rehydration solution (ORS)
E. Rehydration Solution for Malnutrition (ReSoMal)

71. Typical X-ray findings in a patient with tetralogy of fallot (ToF) is


A. Mitralization
B. Fist shaped heart
C. Boot shaped heart
D. Enlarged hilar nodes
E. Globular shaped heart

72. Activated charcoal minimizes absorption in gastrointestinal tract and it is recommended in


which one of the following poisoning?
A. Petrol
B. Ethanol
C. Kerosene
D. Phenobarbitone
E. Hydrochloric acid

73. A child born with birth weight of 2.500kg. at 6 months the child was 5.500kg and t 1 year the
child weighs 8.500kg. the growth curve for this child shows:
A. Normal growth
B. Low weight for age
C. Obesity
D. Marasmus
E. Overweight

74. Breast milk is adequate for the baby up to the age of:
A. 3 months
B. 4 months
C. 6 months
D. 8 months
E. 10 months

75. The characteristics of cerebral spinal fluid drawn from a patient with septic meningitis
contains:
A. High level of sugar
B. Low protein level
C. Normal level of sugar
D. High protein level
E. Normal protein level

CMT BANK OF QUESTIONS Page 14


REVIEW QUESTIONS PAEDIATRIC

76. The following is the signs of good attachment during breastfeeding:


A. Mouth wide open
B. Lower lip turned in
C. Lip pushed forward
D. Chin not touching the breast
E. More areola visible below infant mouth

77. The commonest causative organism of pneumonia in neonate is


A. Group B streptococci
B. Pneumocystis jirovecii
C. Haemophilus influenzae
D. Streptococci pneumoniae
E. Mycoplasma pneumonia

78. Regarding HIV transmission in pediatric, the following rate occur during pregnancy
A. 5%-10%
B. 5%-15%
C. 10%-20%
D. 15%-20%
E. 20%-25%

79. A seven years old boy has swelling of the right upper jaw for 4 weeks. The swelling has been
increasing gradually. However, there is no history of fever associated with swelling. On
examination, the teeth in upper jaw are loose. This child is likely to have
A. Cervical abscess
B. Dental abscess
C. Maxillary osteomyelitis
D. Burkitt’s lymphoma
E. Tuberculosis adenitis

80. The commonest form of sickle cell crises is;


A. Aplastic
B. Haemolytic
C. Sequestration
D. Vaso-occlusive
E. Acute chest syndrome

81. Cotrimoxazole as a prophylaxis of PCP in an HIV exposed neonate is initiated at which age?
A. From the first week of birth
B. From the second week of birth
C. From the third week of birth
D. From the fourth week of birth
E. From 12 weeks

CMT BANK OF QUESTIONS Page 15


REVIEW QUESTIONS PAEDIATRIC

82. A 4 years old boy was suffering from renal disease for more than two years. On examination
he was found to be pale. What will be the most likely type of anemia does the child have?
A. Microcytic hypochromic anaemia
B. Macrocytic normochromic anaemia
C. Normocytic normochromic anaemia
D. Microcytic normochromic anaemia
E. Macrocytic hypochromic anaemia

83. The most common cause of bronchiolitis in neonate is


A. Adenovirus
B. Respiratory syncytial virus
C. Rhinovirus
D. Epstein – Barr Virus (EBV)
E. Myxovirus

84. The following are the minor criteria for diagnosis of Rheumatic fever. EXCEPT
A. St. Vitus dance
B. Arthralgia
C. Fever
D. Leukocytosis
E. Prolonged PR interval on ECG

85. A 3 weeks old female baby presents with difficult breathing and failure to thrive, on
examination systolic murmur was heard on the left lower sternal angle. The doctor diagnosed
congenital heart disease (CHD). What will be the most likely type of CHD?
A. Patient ductus arteriosus
B. Atrial stenosis
C. Aortic stenosis
D. Ventricular septal defect
E. Coarctation of aorta

86. A child is said to have fast breathing if:


A. 2 days old and RR is 58 breaths per minute
B. 18 months old and RR is 40 breaths per minute
C. 5 months old and RR is 46 breaths per minute
D. 4 years old and RR is 36 breaths per minute
E. 9 months old and RR is 46 breath per minute

87. A 7 years old boy went to the hospital after history taking and physical examination the doctor
decides to order full blood picture and results shows macrocytic anaemia. The most likely
infection to this child will be:
A. Plasmodium falciparum
B. Diphyllobothrium latum
C. Hook worm infection

CMT BANK OF QUESTIONS Page 16


REVIEW QUESTIONS PAEDIATRIC

D. Viral hepatitis
E. Ascaris lumbricoides

88. Which of the following is found in category III of TB treatment?


A. New sputum smear negative and less severe form of EPTB
B. New sputum smear, positive PTB and severe form of EPTB
C. Chronic cases and smear positive pulmonary tuberculosis
D. Relapse, treatment failure and sputum smear positive return after default
E. Treatment failure and sputum smear positive return after default

89. A 14 days old male bay was diagnosed to have pneumonia. The most likely causative organism
will be:
A. Adenovirus
B. S.pneumonia
C. Group A streptococcus
D. H.Influenza
E. Group B streptococci

90. The first regimen antiretroviral therapy (ART) in TB/HIV co-infections to a child….years old
is:
A. AZT+3TC+NVP
B. ABC+3TC+NVP
C. AZT+3TC+EFZ
D. d4T+3TC+FDC
E. d4T+3TC+NVP

91. The following are the medicine used as a prophylaxis in a child born from a mother who has
tuberculosis;
A. Rifampicin
B. Pyranamide
C. Ethambutol
D. Isoniazid
E. Streptomycin

92. A one-year-old boy had hot water splash while playing in the kitchen. He sustained second
degree burns on half of his right thigh and whole of the back. Child’s weight is 15kg. The
amount of fluid required by the child in the first 8(eight) hours is:
A. 150ml
B. 180ml
C. 280ml
D. 480ml
E. 960ml

93. Where is the best site to perform ascites tap:

CMT BANK OF QUESTIONS Page 17


REVIEW QUESTIONS PAEDIATRIC

A. Right iliac fossa


B. Left inguinal region
C. Right lumbar region
D. Left Flanks
E. Right hypochondriae region

94. Among the following is one way of assessing child growth


A. Parent’s lifestyle
B. Child’s servings
C. Mid upper arm circumference
D. Weighing the child’s liver
E. How far the child an walk

95. One of the following is found in paroxysmal stage of whooping cough


A. Rhinorrhoea
B. Wheezing
C. Vomiting after coughing
D. Lacrimation
E. Persistent chronic cough

96. A three days old infant is brought to the health center because has developed mucous drainage
from the nose. The discharge was initially watery and had become thick and purulent. Further
inquiry reveals that the child was born at home. There was desquamation of skin on palms.
The child is most likely suffering from:
A. Congenital cytomegalovirus (CMV) infection
B. Congenital Rubella
C. Severe septicemia
D. Congenital Epstein-barr infection
E. Congenital treponema pallidum infection.

97. Administration of dry 100% oxygen can lead to:


A. Hyperventilation of lungs
B. Blindness
C. Increased permeability of pulmonary vessel
D. Alveolitis
E. Bronchiolitis

98. A 3 years old girl was brought to the hospital with her mother with history of cough and
difficulty in breathing; her mother also say her child is unable to drink. On examination there
was lower chest in drawing. What will be the classification of pneumonia to this child?
A. Severe pneumonia
B. Pneumonia
C. Very severe pneumonia
D. Cough or cold

CMT BANK OF QUESTIONS Page 18


REVIEW QUESTIONS PAEDIATRIC

E. Severe pneumonia with wheezing

99. Neonatal sepsis 7 days after birth can be attributed by:


A. Premature rupture of membranes
B. Low birth weight
C. Meconium aspiration
D. Hands of care-provider
E. Maternal fever

100.Signs of good attachment to breast feeding includes:


A. Lips pushed forward
B. Nose touching the breast
C. Lower lip turned in
D. Mouth wide open
E. More areola visible below infants mouth.

101.McKau a 2-years-old boy who weighs 13kg sustained a burn injury. The maintenance fluid
from the first 10 kilograms will be:
A. 150mls in 24hours
B. 500mls in 24hours
C. 1000mls in 24 hours
D. 1050mls in 24 hours
E. 1300mls in 24 hours

102.Steven, a 2-months-old child is brought to a dispensary due to history of poor feeding and
recurrent episodes of seizures. He has history of yellow coloration of body 12 hours after birth.
In physical examinations, he was found to be hypotonic. What is the mostly likely diagnosis?
A. Neonatal sepsis
B. Bilirubin encephalopathy
C. Cerebral palsy
D. Neonatal jaundice
E. Haemolytic deseases of newborn

103.A neonate born five minutes ago exhibits the following signs, pulse rate of 80 beats/minutes,
cyanotic extremities, normal muscle tone, irregular breathing and is weak. What is the
APGAR score?
A. 4
B. 5
C. 6
D. 7
E. 8

104.Walking reflex

CMT BANK OF QUESTIONS Page 19


REVIEW QUESTIONS PAEDIATRIC

A. Is absent at birth because an infant cannot support their own weight


B. Disappear at six weeks due to increase ratio of leg weight to strength
C. It reappears around two years as voluntary behavior
D. It reappears around eight months to one year as a result of involuntary behavior
E. Injury to brachial plexus can cause unilateral absence of the reflex

105.You examine a newborn who delayed crying. On examination: Bluish on both upper and lower
limbs, PR 86bpm, weak response to stimulation, some flexion of the extremities and irregular
respiratory effort. What is the APGAR score?
A. 4
B. 5
C. 6
D. 7
E. 8

106.A 6-year-old boy was brought to the hospital with the history of sudden onset of cough and
difficulty in breathing. His fellow reported that he was eating groundnuts before the onset of
symptoms. What is the most likely diagnosis?
A. Bronchiolitis
B. Acute bronchitis
C. Bronchial asthma
D. Severe pneumonia
E. Foreign body aspiration

107.One of the following is major criteria in the diagnosis of rheumatic fever


A. Fever
B. Carditis
C. Arthralgia
D. Leukocytosis
E. Raised ESR

108. A child with diarrhea and having some dehydration can be treated with
A. ReSoMal
B. Extra feeding
C. Zinc supplement
D. ORS and Zinc supplement
E. Intravenous Ringer’s lactate

109.One of the following is a contraindication to lumber puncture in children


A. History of convulsion
B. Spinal defects at thoracic region
C. A new born with poor suckling reflex
D. A child with history of unconsciousness
E. Signs of increased intracranial pressure

CMT BANK OF QUESTIONS Page 20


REVIEW QUESTIONS PAEDIATRIC

110.The proper management of child with history of bleeding at the site of injection after
intramuscular injection is: -
A. IM Vitamin K1 to 2mg stat
B. Blood transfusion
C. Haematenics
D. I.V. fluid - normal saline
E. Apply direct pressure to the bleeding site.

111.A 3-day neonate was brought to a dispensary with complaints of yellowish discoloration of
skin for one day. On examination: she is alert, afebrile, jaundiced, with normal suckling and
mororeflex. What is the mostly likely diagnosis?
A. Haemolytic disease of newborn
B. Birth asphyxia
C. Neonatal septicemia
D. Pathological jaundice
E. Physiological jaundice

112.A 12 months old boy was brought to the health center with complaints of cough, difficulty in
breathing and in ability to breastfeed for 1 day. On examination: febrile (temperature =
38.6oC), dyspnoeic, cyanotic, with lower chest in drawing. What is the most likely diagnosis?
A. Septicemia
B. Acute bronchitis
C. Bronchiolitis
D. Severe pneumonia
E. Pulmonary tuberculosis

113.One of the following is an emergency sign


A. Lethargy
B. Severe pain
C. Convulsion
D. High grade fever
E. Severe palma pallor

114.One of the common side effect of Zidovudine in HIV patients includes:


A. Elevated liver enzymes
B. Severe skin rashes
C. Bluish discolouration of finger nails
D. Sleep disturbance
E. Exfoliative dermatitis.

115.A 6-year-old boy was brought to the health center with history of severe joint pain; his mother
reported that her child had a history of several blood transfusions. On examination he was
pale, jaundiced and boozing of the skull. What is the most likely diagnosis?
A. Leukemia

CMT BANK OF QUESTIONS Page 21


REVIEW QUESTIONS PAEDIATRIC

B. Osteomyelitis
C. Rheumatic fever
D. Sickle cell disease
E. Rheumatoid arthritis

116.You are called to review a new born baby 12 hours post-delivery who presents with yellowish
discoloration of the body. On further questioning, you found that her mother was blood group
Rh negative. On examination; the new born was pale and febrile. What is the most likely
diagnosis?
A. Neonatal sepsis
B. Neonatal jaundice
C. ABO incompatibility
D. Hemolytic disease of newborm
E. Hemorrhagic disease of newborn.

117.A newborn present with a history of failure to cry immediately after delivery. On examination
floppy, cyanotic and APGAR score was 5 at 5th minutes. What is the most likely diagnosis?
A. Delay cry
B. Severe pneumonia
C. Severe birth asphyxia
D. Respiratory distress syndrome
E. Cyanotic congenital heart disease

118.A 4-year old body weighing 20kg was brought to the dispensary with a 3 days history of
watery diarrhea. He has an average of 6 motions per day. On examination: Lethargic, sunken
eyes and skin pinch goes back very slowly. What type and amount of fluid you will give the
boy in the first 30 minutes?
A. 1400mls of Ringer’s Lactate
B. 600mls of 5% Dextrose
C. 1400mls of Oral Rehydration Salt
D. 600mls of Ringer’s Lactate
E. 600mls of Oral Rehydration salt.

119.Which of the following poisons can be managed by gastric lavage?


A. Acid products
B. Alkali products
C. Mushroom
D. Organophosphate
E. Petroleum products

120.A 3years old HIV positive girl is brought to a dispensary with clinical features suggestive of
HIV clinical stage III. Which of the following support clinical staging?
A. Papular pruritic eruption
B. Oral hairy leukoplakia

CMT BANK OF QUESTIONS Page 22


REVIEW QUESTIONS PAEDIATRIC

C. Pneumocystis pneumonia
D. Herpes zoster
E. Fungal nail infection.

121.A 2-year-old boy weighing 15kgs sustained burn injury on the anterior part of the chest and
abdomen following hot water splash. What amount of fluid that should be given for
resuscitation in the first 24 hours?
A. 780mls of Ringer’s Lactate
B. 680mls of Ringer’s Lactate
C. 600mls of Ringer’s Lactate
D. 390mls of Ringer’s Lactate
E. 195mls of Ringer’s Lactate

122.While performing an assessment of level of consciousness of an 11 months old infants by


using the Blantyre coma scale, the attending clinical assistant observed that, the baby
withdraws from painful stimuli, follows objects and has abnormal cry. What will be the score?
A. 1
B. 2
C. 3
D. 4
E. 5

123.Rapid respiratory rate in a 1-week-old baby is:


A. 30 breaths per minute and above
B. 40 breaths per minute and above
C. 50 breaths per minute and above
D. 70 breaths per minute and above
E. 60 breaths per minute and above

124.With regard to anthropometric measurement, microcephaly is correctly describe in terms of:


A. Weight for age
B. Height/length for age
C. Height/length for weight
D. Mid upper arm circumference
E. Occipito-frontal circumference

125.During triage you found a child in coma. The next immediate step is:
A. Start giving oxygen preferably 1-2 litres per minute
B. Start chest compression about 40 compressions per minute
C. Insert IV line and start giving IV fluids like lactate a Ringer
D. Look for any head/neck trauma before positioning a child
E. Insert nasogastric tube and urinary catheter for monitoring of output

CMT BANK OF QUESTIONS Page 23


REVIEW QUESTIONS PAEDIATRIC

126.The following drug should NOT be given to the 5-day-old baby


A. Chloramphenicol
B. Paracetamol
C. Ceftriaxone
D. Amoxicillin
E. Gentamicin

127.The proper feeding advice to a child who is eight months old is


A. Tell her to exclusive breastfed and no complementary food
B. Stop breastfeeding and start giving porridge and mashed food
C. Give two snacks between per day and breastfed about 8 times a day
D. Give family food about 3 times a day and breastfeed about 8 times a day
E. Start giving thick enriched porridge and mashed food together with breast milk

128. A 5 years old boy presents to dispensary due to confusion over several hours. His mother
reported several days history of increased thirsty, weight loss, frequency day and time and
nocturnal urination. On examination: afebrile, dehydrated with kussmaul breathing, and
MRDT negative. What is the most likely diagnosis?
A. Coma
B. Shock
C. Hyperglycemia
D. Obstructed breathing
E. Diabetic ketoacidosis

129.Which of the following tools used in screening newborn babies for congenital heart disease?
A. Audiogram
B. Filter paper
C. Pulse oximeter
D. Echocardiogram
E. Electrocardiogram

130.A 2-year-old boy presents at dispensary with loss of consciousness after sustaining bee sting
1 hour ago. On examination: Unconscious, cold extremities, delayed capillary refill, weak and
fast pulse, weight 12kg. The most appropriate management of this child should include:
A. IV Ringer’s lactate 240mls to run fast, chlorpheniramine and blood transfusion
B. IV Normal saline 1200mls to run fast, Hydrocortisone and blood transfusion
C. IV 10% Dextrose 240mls to run fast, chloropheniramine and referral
D. IV Normal saline 240mls to run fast, chloropheniramine and referral
E. IV 10% Dextrose 1200mls to run fast, Hydrocortisone and referral

131.The amount of daily maintenance fluid to be given on the second day to a 3 year old child
weighing 15kgs who sustained 15% body surface are burn is:
A. 1500mls
B. 1250mls

CMT BANK OF QUESTIONS Page 24


REVIEW QUESTIONS PAEDIATRIC

C. 900mls
D. 450mls
E. 300mls

132.A 6-year-old boy presented with complains of left upper jaw swelling for 6 months which is
associated with left eye protrusion and weight loss. On examination: Afebrile, wasted,
Exophthalmos, swelling of left maxilla, and loosening teeth. Which infectious organism is
commonly associated with this condition?
A. Human papilloma virus
B. Trypanosome species
C. Herpes simplex virus
D. Helocobacter pylori
E. Epistein Bar virus

133.A 9-months old child who weighs 10kg has been rushed into the consultation room after
developing convulsion while waiting in the queue. What is the most appropriate management
of this child?
A. Tablet diazepam 5mg stat
B. Per rectal diazepam 5mg stat
C. IV phenobarbitone 150mg stat
D. Tablet phenobarbitone 150mg stat
E. Per rectal phenobarbitone 150mg stat

134. A 5-year-old boy presents to dispensary complaining of passing cola coloured urine
associated with facial swelling which tend to be more severe in the morning. On examination:
alert, afebrile, puffy face and elevated blood pressure. What is the most important parameter
in urinary analysis for this patient?
A. Epithelial cells
B. Pus cells
C. Cystrals
D. Nitrites
E. Protein

135.A clinician is called to review a 4-year-old child who has been unconscious for the past one
day. On examination: unconscious, secretions in the mouth and cyanosis. What is the initial
step in management of this child?
A. Suctioning
B. Oxygen therapy
C. Bag mask ventilation
D. Heumlich’s Maneuver
E. Left lateral positioning

136.A sickle cell child presenting with priapism is in risk of developing which of the following
complication?

CMT BANK OF QUESTIONS Page 25


REVIEW QUESTIONS PAEDIATRIC

A. Sterility
B. Balanitis
C. Impotence
D. Epispadiasis
E. Cryptorchidism

137.The WHO Pediatric clinical staging IV of HIV/AIDS includes:


A. Unexplained anaemia
B. Extensive molluscum contangiosum
C. Unexplained persistent hyper splenomegaly
D. Recurrent or chronic upper respiratory tract infection
E. Unexplained severe wasting, stunting or severe malnutrition

138.A 7-year-old boy has been on treatment for pneumonia for past 4 days with no improvement.
On physical examination the child was dyspneic, rapid heart rate, and fast breathing rate with
bilateral basal crepitation. Abdominal examination reveals a soft enlarged tender liver with
bilateral pitting edema at the ankles. What is the most likely diagnosis?
A. Pulmonary TB
B. Pneumothorax
C. Pleural effusion
D. Pulmonary embolism
E. Congestive heart failure

139.One of the following can be found in patient with nephrotic syndrome:


A. Proteinuria
B. Haematuria
C. Hypertension
D. Hyperlipidemia
E. Hyperalbuminaemia

140.A 8-year-old girl presents to hospital with history of high grade fever and convulsion for one
day. On examination; febrile temperature 39.20C, laboratory findings of the CSF analysis
reveals cloudy CSF, Leukocytosis, reduced glucose level. What is the most likely diagnosis?
A. Septicemia
B. Encephalitis
C. Severe malaria
D. Viral meningitis
E. Bacterial meningitis

141.A 3-year-old brought to the dispensary by his mother complaining of three days history of
profuse watery stool about six episodes per day. On examination: He was irritable, sunken
eyes, skin pinch goes back slowly. What is the most likely diagnosis?
A. Persistent diarrhea
B. Acute chronic diarrhea

CMT BANK OF QUESTIONS Page 26


REVIEW QUESTIONS PAEDIATRIC

C. Acute diarrhea with no dehydration


D. Acute watery diarrhea with some dehydration
E. Acute watery diarrhea with severe dehydration

142. With A 1-year-old boy with 9.5kg presents with acute watery diarrhea and severe
dehydration. The amount of ringer’s lactate that should be given in the first half (½) an hour:
A. 275mls
B. 280mls
C. 285mls
D. 300mls
E. 305mls

143.A 3-year-old girls with 13kg was brought by her mother complaining of hot water splash while
she was playing in the kitchen with her friend. On examination: Blister both the anterior and
posterior part of the trunk, tender on touch. What is an estimated amount of fluid that must be
given during the first 8hours?
A. 676mls
B. 682mls
C. 936mls
D. 1352mls
E. 1872mls

144.One of the following investigations will establish the cause of anaemia


A. Haematocrit
B. Stool for ova
C. Serum ferritin
D. Total iron binding capacity
E. Haemoglobin level estimation

145.Severe form of extra pulmonary TB includes


A. TB adenitis
B. Military TB
C. TB of gland
D. TB of the bone
E. TB of adenoids

146.A 12 hours new born presents with generalized yellowish discoloration of the whole body. On
examination is afebrile (temp 36.20C) and acyanotic what is the most likely diagnosis?
A. Neonatal sepsis
B. Hypoglycemia
C. Congenital malaria
D. Congenital syphilis
E. Pathological jaundice

CMT BANK OF QUESTIONS Page 27


REVIEW QUESTIONS PAEDIATRIC

147.Regarding to the effects of Patent Ductus Arteriosus which of the following statements is
correct?
A. Decrease pulmonary blood flow
B. Increase blood flow to the lungs
C. Increase blood flow to the systemic circulation
D. Results to decrease in diameter of the pulmonary artery
E. Oxygenated blood passes from pulmonary artery to aorta

148.A 4-year-old boy presents with facial edema, on examination and investigation he was found
to have, heavy proteinuria, hypo-albuminemia and hyperlipidemia. What might be the most
likely diagnosis?
A. Pyelonephritis
B. Hyper-lipidemia
C. Nephrotic syndrome
D. Nephritic syndrome
E. Urinary tract infection

149.With regard to clinical features of nephrotic syndrome which among the following statement
is correct?
A. It is common in children below two years old
B. Hypertension is uncommon among these patient
C. It is commonly occurring 1-2 days following acute pharyngitis
D. In severe cases the disease might progress to acute renal failure
E. There is no chance of nephrotic syndrome to progress to nephritic syndrome (vice versa)

150.A 4-day-old infant brought by her mother to the dispensary with clinical features suggestive
to neonatal jaundice. Which of the following is an appropriate management for this patient?
A. Phototherapy
B. MRDT then refer
C. Exchange transfusion
D. Give oxygen and reassure the mother
E. Admit the patient and give IV antibiotics

151.A 6-month-old child was brought at OPD with complaints of fever for 2 days and generalized
tonic-clonic convulsion which started at the OPD, the body weight of the child was 6kg. what
was the correct dose of rectal diazepam?
A. 0.6mg
B. 1.2mg
C. 2.0mg
D. 2.5mg
E. 3.0mg

152.Concerning the diagnosis of paediatric TB. The minimum score for diagnosis of TB using
scoring chart is children is;

CMT BANK OF QUESTIONS Page 28


REVIEW QUESTIONS PAEDIATRIC

A. 6
B. 7
C. 8
D. 9
E. 10

153.Mastoiditis in paediatric often results as the complication of which of the following disease?
A. Otitis media
B. Pneumonia
C. Pharyngitis
D. Meningitis
E. Sinusitis

154.A 2-year-old female child was brought at OPD with complain of loose stool for 2 days. The
stool is not blood stained, the child had about 5 motion for past 24 hours. On examination, the
child was easily irritable, sunken eyes, drinks eagerly and skin pinch goes back slowly. What
is the most likely diagnosis?
A. Dysentery with some dehydration
B. Dysentery with severe dehydration
C. Acute watery diarrhea with no dehydration
D. Acute watery diarrhea with some dehydration
E. Acute watery diarrhea with severe dehydration

155.A 2-year-old child with history of failure to thrive since birth is brought to the hospital with
an acute onset of difficulty in breathing. On examination has central cyanosis with a systolic
murmur on auscultation. The rest of the examination was normal. What is the most likely
diagnosis?
A. Endocarditis
B. Atrial septal defect
C. Mitral valve defect
D. Tetralogy of fallot
E. Ventral septal defects

156.A 9-year-old child attended at the dispensary with the complaint of skin lesion on the hand,
the lesion started one year ago they are non-itching. On examination the lesions are multiple,
hypopigmented with palpable ulnar at the left side of the hand. What is the most likely skin
infection?
A. Scabies
B. Leprosy
C. Psoriasis
D. Ring worm
E. Contact dermatitis

CMT BANK OF QUESTIONS Page 29


REVIEW QUESTIONS PAEDIATRIC

157.A 4-year-old child with body weight of 15kg was brought to the hospital with complaints of
loss of consciousness for 6 hours. There is no history of fever. T-370C , RBG-2mmol/L. Which
of the following is the correct dose of 5% dextrose to be administered to the child?
A. 100mls
B. 150mls
C. 200mls
D. 250mls
E. 300mls

158.Which of the following investigations is used in the diagnosis of HIV infection in children
under 18 months;
A. CD4+ count
B. SD-Bioline
C. Unigold
D. ELISA
E. PCR

159.A 6-months child who was born vaginally at home attended at the dispensary with the
complaints of passing stool stained with blood. Per abdomen examination; sausage shaped
mass was palpated. One of the following is the correct diagnosis
A. Omphalocele
B. Gastroenteritis
C. Intussusception
D. Faecal impaction
E. Hirchsprung disease

160.Which of the following clinical features is consistent with WHO HIV/AIDS classification
stage II?
A. Anaemia
B. Esophageal candidiasis
C. Cyptococcus meningitis
D. Pneumocystis pneumonia
E. Extra pulmonary ruberculosis

161.A 2-year-old brought at the dispensary with complaint of failure to thrive for 2 months and
loose stool for 3 days. The child normally eats on maize porridge about 200mls three times a
day. On examination she drinks normally, weight for age was-2SD. Which of the following is
correct fluid to be given to the child?
A. Resomal
B. IV 5% dextrose
C. IV Ringer lactate
D. IV Normal saline
E. Oral rehydration solution

CMT BANK OF QUESTIONS Page 30


REVIEW QUESTIONS PAEDIATRIC

162.One of the following feature is commonly observed in children with cerebral palsy;
A. Spasticity
B. Paraplegia
C. Sun set eyes
D. Antalgic gait
E. Macrocephaly

163.Which of the following is the criterion used to discharge the patient who has been admitted
due to severe malnutrition?
A. Decrease of edema
B. Increased of weight
C. Good appetite in plampynuts
D. Ability of the child to drink F75
E. Ability of the child to drink F100

164.A 5-year-old girl was brought to the hospital with complaints of sudden onset of difficulty in
breathing and dry cough for 2 hours. There is history of atopic disease in the family. On
examination: Afebrile, dyspneic, Bronchial breathing sound with wheezes heared. Which of
the following drugs should be included in management for reducing inflammation?
A. Hydrocortisone
B. Ceftriaxone
C. Salbutamol
D. Albuterol
E. Oxygen

165.Regarding organism which cause malaria in children, one among the following attacks both
young red blood cells and adult red blood cells
A. Plasmodium ovale
B. Plasmodium vivax
C. Plasmodium malariae
D. Plasmodium falciparum
E. Plasmodium knowleski

166.Which of the following is the correct age at which the anterior fontanele closed in children?
A. At 6 months
B. At 12 months
C. At 18 months
D. At 24 months
E. At 30 months

167.A 10-year-old girl is brought to the hospital with complaints of high grade fever for three days.
On examination the child is responding to verbal, kerning sign was positive, brudzinski sign
was negative. Which of the following is the confirmatory investigation?
A. Skull X-ray

CMT BANK OF QUESTIONS Page 31


REVIEW QUESTIONS PAEDIATRIC

B. CSF analysis
C. Brain CT scan
D. Full blood picture
E. Electroencephalogram

168.A 7-year-old boy was brought to the health center with complain of abdominal mass which
was accidentally found by the mother. On examination; He was moderately pale, jaundice,
not dyspneic. Per abdomen; there was asymmetrical abdominal distension with nodulated and
non-tender mass on the left upper quadrant. One of the following is the correct diagnosis:
A. Hepatoma
B. Lymphoma
C. Neuroblastoma
D. Nephroblastoma
E. Bladder tumor

169.One of the following statements correctly describe the clinical manifestation of Sickle cell
disease
A. Priapsim can lead to future impotence
B. Iron deficiency in major cause of anaemia
C. Rarely, painful crisis is due to vaso-occlusion
D. Sequestration crisis is common than painful crisis
E. Osteomyelitis is often due to streptococcal infection

170.A 4-year-old child was admitted to the hospital with the complaints of altered level of
consciousness and difficulty in breathing for one day. On examination he was not pale,
dyspneic, kussmaul’s hyperventilation with acetone smell. The rest of the examination was
normal. Most likely findings in laboratory investigation is?
A. MRDT positive
B. Increased urine ketone
C. Low serum glucose level
D. Increased serum creatine level
E. Decreased white blood cell count

171.The following is a clinical feature of a child in shock


A. High fever
B. Convulsion
C. Tented skin
D. Weak and fast pulse
E. Lower limb oedema

172.One of the following is the most likely cause of pathological neonatal jaundice
A. Biliary atresia
B. Congenital malaria
C. Congenital syphilis

CMT BANK OF QUESTIONS Page 32


REVIEW QUESTIONS PAEDIATRIC

D. Vitamin K deficiency
E. Congenial Heart Disease

173.Known diabetic child, 10-year-old presented to the hospital with loss of consciousness for 2
hours and had acetone smell. Which one of the following investigations help to confirm the
diagnosis:
A. Malaria rapid diagnostic test
B. Urine dipstick for ketones
C. Hemoglobin estimation
D. Fasting blood glucose
E. Brain CT-scan

174.A 12-year-old girl presented at the OPD with the complaints of joints pains, awareness of
heartbeat beat for 1 week. Her mother reported of previous history of several blood
transfusions due to similar condition. Which of the following is the most likely diagnosis:
A. Heart failure
B. Severe malaria
C. Rheumatic fever
D. Sickle cell disease
E. Rheumatoid arthritis

175.One of the following is clinical feature for WHO HIV stage 3 in pediatrics:
A. Extra pulmonary Tuberculosis
B. Persistent oral candidiasis
C. Oesophageal candidiasis
D. Kaposis sarcoma
E. Herpes zoster

176.A newborn is admitted to the neonatology unit with bleeding gums. On examination; the child
was pale, prolonged prothrombin and bleeding time. The most likely cause of this cause of
this baby condition is;
A. Anaphylactic reaction
B. Intrauterine infections
C. Surfactant deficiency
D. Vitamin K deficiency
E. Birth trauma

177.One of the following is the correct management of a child with hypoglycemia at the dispensary
A. IV 10% dextrose 10mls/kg for 10 minutes
B. IV 10% dextrose 50mls/kg for 30 minutes
C. IV 10% dextrose 5mlskg as bolus
D. IV 10% dextrose 20mls/kg rapidly
E. IV 10% dextrose 1ml/kg rapidly

CMT BANK OF QUESTIONS Page 33


REVIEW QUESTIONS PAEDIATRIC

178.One among the following is an emergency sign in pediatrics triaging;


A. Vomiting everything
B. Respiratory distress
C. High grade fever
D. Chest in drawing
E. Fast breathing

179.Drug of choice for rheumatic fever prophylaxis in patient who are allergic to penicillin is;
A. Ceftriaxone
B. Tetracycline
C. Gentamycin
D. Clindamycin
E. Erythromycin

180.A 6-year-old child is admitted to the pediatric ward with complaints of cough, fever and
difficulty in breathing for one day. On examination: the patient has lower chest in drawing,
central cyanosis and on auscultation crackles were audible. The appropriate immediate
intervention is;
A. Administering oxygen
B. Open airway by neck extension
C. Give high dose of hydrocortisone
D. Start cardio-respiratory resuscitation
E. Put the child in prone and perform

181.A child who is well nourished, which is the value of random blood glucose for hypoglycemia
to be diagnosed?
A. 2.5mmol/l
B. 3.5mmol/l
C. 5.0mmol/l
D. 7.0mmol/l
E. 11.1mmol/l

MULTIPLE CHOICE TRUE/FALSE QUESTIONS


1. Regarding fluid management in Burn patient
A. TRUE There is fluid loss in burned area so extra fluid is needed
B. FALSE I/V fluid is indicated if burned surface area is less than 10% burn
C. TRUE Ringers lactate solution is the fluid of choice
D. TRUE Children are more susceptible to dehydration
E. FALSE Dextrose normal saline is the fluid of choice

CMT BANK OF QUESTIONS Page 34


REVIEW QUESTIONS PAEDIATRIC

2. The following are signs of good attachment for breastfeeding


A. FALSE Lips pushed forward
B. TRUE Chin touching the breast
C. FALSE More areola visible below the infant’s mouth
D. TRUE Mouth wide open
E. TRUE Lower lip turned out

3. The following are the common side effects of lamivudine


A. FALSE Anaemia
B. FALSE Anorexia
C. TRUE Insomnia
D. TRUE Headaches
E. FALSE Mylgia

4. Regarding diagnosis and treatment of short-stature


A. TRUE One need to obtain familial history of inherited skeletal dysplasia
B. FALSE Chronic renal disease is the most common cause
C. FALSE With normal variant short stature, treatment is necessary
D. TRUE Medical treatment depends on cause of stature
E. TRUE Stadiometer which is well calibrated needs to be used in examining height

5. The following statements are true regarding measles


A. TRUE Rashes appears 14 days after exposure
B. TRUE Prodromal phase is characterized by cough, coryza and conjuctivitis
C. TRUE Malnutrition is one of the complication
D. FALSE Incubation period is 6-8 days
E. FALSE Left thigh is the is the site of injection of the vaccine

6. The following localized signs and symptoms may suggest tuberculosis


A. TRUE Abdominal swelling ascites
B. TRUE Febrile illness with pleural pain and effusion
C. TRUE Chronic painful swelling in a weight-bearing joint
D. TRUE Stiff painful back, spinal deformity and paraplegia
E. FALSE Acute swelling of lymph nodes with or without drainage

7. Concerning prevention methods for measles


A. TRUE Do not postpone immunization because of moderate current illness
B. FALSE Incase of measles contact, immunize non-immunized children below 6 months
on admission
C. TRUE Ensure good nutrition including vitamin A rich food
D. TRUE Isolate active cases of measles where possible, especially from
immunocompromised individual
E. FALSE Five days before and after rash the child is still infectious.

CMT BANK OF QUESTIONS Page 35


REVIEW QUESTIONS PAEDIATRIC

8. Corticosteroids are beneficial in treatment of which form of tuberculosis:


A. FALSE Tuberculosis adenitis
B. TRUE Milliary Tuberculosis
C. TRUE Tuberculosis laryngitis
D. TRUE Tuberculosis Meningitis
E. FALSE Tuberculosis Osteomyelitis

9. Regarding neonatal jaundice


A. FALSE Appearance of jaundice in neonatal period is pathological
B. TRUE Jaundice appearing on the first day of life is pathological
C. TRUE Physiological jaundice is caused by immaturity of liver cells
D. TRUE Hyperbilirubinemia
E. FALSE Bilirubin level in pathological jaundice is less than 12mg/dl

10. Feature of WHO Health organization (WHO) Paediatric Clinical stage 4 of HIV and AIDS
A. FALSE Unexplained persistent diarrhea
B. FALSE Pulmonary tuberculosis
C. TRUE TB of the spine
D. TRUE Cryptococcal mengingitis
E. TRUE Kaposi’s sarcoma

11. Urinalysis findings that suggest urinary tract infection:


A. TRUE More than 5WBC/hpf in uncentrifuged urine
B. TRUE More than 15WBC/hpf of centrifuged urine
C. TRUE Presence of protein
D. FALSE Presence of epithelia cells
E. FALSE Presence of calcium salts

12. Criteria for initiating antiretroviral therapy (ART) in infant below 18 months:
A. TRUE All HIV infected infants below 12 months of age with virological proven infection
B. TRUE All HIV exposed infants under age of 12 months
C. FALSE All HIV exposed infants aged 12-18 months who are WHO stage 1 of 2 with
CD4 greater than 20%
D. TRUE All HIV exposed infant below 18 months without virologocal confirmation in
WHO clinical stage 3 or 4
E. FALSE All HIV exposed infants below 18 months with positive antibody test.

13. The following are the characteristics of type 2 diabetes mellitus


A. TRUE Age more than 10 years
B. FALSE Classical symptoms of polyuria and polydipsia
C. TRUE Overweight
D. FALSE Dehydration
E. TRUE Family history

CMT BANK OF QUESTIONS Page 36


REVIEW QUESTIONS PAEDIATRIC

14. Management of otitis media in children includes


A. TRUE Ephedrine nasal drops to relieve obstruction of Eustachian Tube
B. FALSE Antibiotic ear drops are advocated
C. TRUE Ear syringing is very effective
D. FALSE Myringotomy is sometimes indicated
E. FALSE Ear wicking is indicated in presence of purulent discharge

15. Blood transfusion is indicated to


A. TRUE Acute blood loss
B. FALSE Sickle cell anemia
C. TRUE To provide clotting factors
D. FALSE All cases of septic shock
E. TRUE Any child with severe anaemia

16. Management of rectal prolapse at the health center involves


A. TRUE Reduction with gentle pressure
B. FALSE Referral of all cases
C. TRUE Strapping of buttocks following reductions
D. TRUE Treatment of diarrhea and malnutrition
E. FALSE Giving broad spectrum antibiotics

17. Signs of significant prematurity include:


A. TRUE Absence of palmer skin creases.
B. FALSE Testes are palpable in the scrotum with scrotal rugae visible
C. TRUE Flat pinnae of the ears.
D. TRUE Absence of palpable breast tissue.
E. FALSE Clitoris and labia minora visible with difficulties.

18. In louse infestations


A. FALSE Body louse is known as Phthrus pubis
B. TRUE Severe itching occurs
C. FALSE Lymphadenopathy is not a feature
D. TRUE Eggs appears grey white specks
E. FALSE Lice live and lay eggs in the soil

19. Taenia solium


A. FALSE This is a beef tapeworm
B. TRUE Usually asymptomatic when larva is ingested
C. TRUE Eggs hatch in the small intestines and cause cysticercosis
D. TRUE May cause blindness
E. TRUE Focal neurological deficit

CMT BANK OF QUESTIONS Page 37


REVIEW QUESTIONS PAEDIATRIC

20. The immediate management of a 6-year old child presenting with breathlessness of recent
onset associated with tachycardia, gallop rhythm, systolic murmur, tachypnea and tender
hepatomegaly includes
A. TRUE Putting the child in a supine position to reduce the workload of the heart
B. TRUE Giving supplemental oxygen
C. FALSE Administer furosemide for its vasodilating effects as well as diuretic effects
D. FALSE Administering digoxin for its positive inotropic effects on the heart
E. TRUE Avoiding nitrates which have a negative inotropic effect on the heart

21. A 20-month old with Teratology of fallot is admitted for evaluation of cyanosis that is
increasing in frequency. As you start the examination you witness an episode of cyanosis and
as the crying increases, the patient becomes more and more cyanotic. What is the next most
appropriate step in the management of the child?
A. TRUE Initiate and begin mechanical ventilation
B. FALSE Administer IV dopamine
C. TRUE Place the patient in a knee-chest position
D. FALSE Administer subcutaneous adrenaline
E. TRUE Call for cardiological consultation

22. The following paediatric conditions need referral:


A. TRUE Neonatal sepsis
B. TRUE Low birth weight
C. TRUE Foreign body in the airways
D. FALSE 5% BSA burn
E. TRUE Birth asphyxia

23. Regarding clinical presentation of a child with burn injury:


A. TRUE Charred wound indicate involvement of tissue underneath skin.
B. FALSE Painless lesion indicate involvement of derms.
C. TRUE Blister with red underground suggest involvement of epidermis.
D. FALSE Black lesion is a sign of partial thickness burn.
E. TRUE Painful burn lesion suggests the involvement of epidermis.

24. Facts regarding the vulnerability of children to various injuries:


A. TRUE Children with small airway obstruction are at greater risk of aspiration.
B. TRUE Infant burn more deeply and quickly at lower temperature.
C. FALSE Children with small stature are at risk of chest injuries
D. FALSE Grasping behavior expose children to fractures
E. TRUE Children born to families with low social-economic status.

25. Helen, a 6 years old girl has been asthmatic since 3 years of age. She was rushed to OPD with
a severe attack. The doctor on call was suspicious of other diagnosis like
A. TRUE Foreign body
B. TRUE Vascular ring

CMT BANK OF QUESTIONS Page 38


REVIEW QUESTIONS PAEDIATRIC

C. FALSE Intestinal obstruction


D. TRUE Mediastinal mass
E. FALSE Renal stone

26. Bacterial meningitis can be treated with the following regime


A. TRUE Ampicillin and Chloramphenicol
B. TRUE Cephalosporin 3rd generations only
C. TRUE Penicillin G and Chloramphenicol
D. TRUE In cerebral edema elevate the head about 30 degrees
E. TRUE IV fluid may be needed during the first days

27. Regarding Moro reflex: -


A. TRUE Is absent in adults as a results of development of frontal lobes
B. TRUE Moro reflex is present at birth and may last up to six months of age
C. TRUE Damage to infant’s central nervous system can cause bilateral absence of reflex
D. FALSE Fracture of the clavicle can cause bilateral absence of the reflex
E. TRUE Birth trauma can cause unilateral absence of the reflex.

28. Regarding clinical presentation of acute flaccid paralysis caused by poliomyelitis


A. FALSE In abortive form- patient will present with tripod signs i.e. Positive kerning,
Brudzinski sign and bulging fontanelle.
B. TRUE In encephalitic form – the patient will be irritable, drowsy and disoriented.
C. TRUE In paralytic form – the patient will present with constipation, paresthesia and
difficult in swallowing
D. FALSE In bulbar form – the patient will present with facial and extremity weakness that
may involve intercostal muscles and diaphragm.
E. TRUE In non-paralytic form- the patient will present with neck stiffness and sign of
meningeal irritation.

29. Which of the following features indicate severe malaria in children?


A. TRUE Oliguria
B. FALSE Blood glucose = 3mms/L
C. TRUE Haematocrit < 21%
D. TRUE Lactic acidosis
E. TRUE Algid malaria

30. Concerning type two (II) Diabetes Mellitus


A. TRUE The risk is increased with obesity
B. FALSE It is an insulin dependent
C. FALSE Genetic determinant paly role in its susceptability
D. TRUE It is associated with acanthosis nigricans
E. TRUE Suspected if age is greater than 10 years.

31. Which among of the following are complications of sickle cell disease:

CMT BANK OF QUESTIONS Page 39


REVIEW QUESTIONS PAEDIATRIC

A. TRUE Pripism
B. TRUE Stroke
C. FALSE Hypertension
D. TRUE Renal detachment
E. FALSE Pulmonary embolism

32. Concerning pathological jaundice:


A. TRUE It occurs within 24 hours.
B. FALSE Occurs after 24 hours.
C. TRUE Treat with phototherapy if total serum bilirubin is > 350mmol/L
D. TRUE Kernicterus is one of the complications
E. FALSE Exchange transfusion is the only treatment of choice.

33. Which among of the following are the fetal causes of Asphyxia neonatorum:
A. TRUE Birth injuries
B. FALSE Drug like halothane
C. TRUE Respiratory distress syndrome
D. FALSE Diabetes Mellitus
E. TRUE Intra uterine growth retardation

34. The following are the danger signs in neonate


A. TRUE Respiratory rate less than 20 breaths per minute
B. TRUE Drowsy
C. FALSE Depressed anterior fontanelle
D. FALSE Peripheral cyanosis
E. TRUE Grunting

35. Regarding perinatal causes of cerebral palsy:


A. FALSE TORCHES - Infection
B. TRUE Intracranial bleeding
C. FALSE Malformation of brain
D. TRUE Ante partum haemorrhage
E. FALSE Cerebral malaria

36. Regarding feeding recommendation of a child less than 6 months:


A. FALSE Breastfed as often as child wants day and night at least 6 times in 24 hours
B. TRUE Child with low birth weight feed at least every 2-3hours
C. FALSE Give food and fluid as child wants
D. TRUE Add complementary food for a child who is more than 4 months and appears
hungry after breastfeeding and not gaining weight.
E. TRUE Exclusive breast feeding is recommended from birth until 6 months of age.

37. Mwajuma delivered a full term baby by assisted vaginal delivery. The baby cried 5 minutes
later after effective resuscitation. The following may be causes of low score in one minute

CMT BANK OF QUESTIONS Page 40


REVIEW QUESTIONS PAEDIATRIC

A. FALSE Heavy sedation


B. TRUE Fetal distress
C. TRUE Brain injury
D. FALSE Inadequate artificial respiration
E. TRUE Severe congenital malformation

38. Regarding intestinal obstruction in children:


A. TRUE Proximal obstruction present with vomiting with minimal distension
B. TRUE Distal obstruction present with distension with vomiting occur later
C. FALSE The pain is typically burning in nature with no flatus
D. TRUE Intussusceptions usually occurs in children less than 2 years old
E. TRUE Pain is typical cramping in nature with no flatus

39. The following are the appropriate cord care practices


A. FALSE Keep the umbilical cord stump lightly covered with clean clothes
B. TRUE Wash hand before and after cord care
C. TRUE Do not bandage the umbilical cord stump on abdomen
D. FALSE Clean with antiseptic on the umbilical cord stump
E. FALSE Wash hands only after cord care

40. The following are the localized signs and symptoms may suggest tuberculosis:
A. TRUE Febrile illiness with pleural pain and effusion
B. TRUE Chronic swelling of lymph nodes with or without drainage
C. FALSE Abdominal swelling due to splenomegaly
D. TRUE Signs of space occupying intracranial lesion
E. FALSE Painful micturition associated with painful haematuria

41. In the treatment of acute glomerulonephritis, the following are considered


A. TRUE Eradicate residual streptococcal infection by penicillin
B. TRUE Treat hypertension
C. FALSE High protein diet is recommended as part of treatment
D. TRUE Sodium and Fluid restriction should be advised
E. FALSE Prednisolone is absolutely contraindicated

42. The following should considered when treating sickle cell crises
A. TRUE Plenty of IV fluids preferably normal saline or Ringer’s Lactate
B. TRUE Analgesics for painful crisis
C. FALSE Prednisolone is used for treatment of osteomyelitis
D. FALSE Blood transfusion is indicated in severe painful crisis
E. TRUE Refer patient to hospital if is in haemolytic crisis.

43. Requirement for treatment of Croup syndrome are:


A. TRUE Maintain airway
B. TRUE Give nebulizer adrenaline

CMT BANK OF QUESTIONS Page 41


REVIEW QUESTIONS PAEDIATRIC

C. TRUE Keep the child comfortable in semi-seated position


D. FALSE Start IV antibiotics immediately
E. FALSE Isolate the baby to avoid transmission of the disease

44. Induction of vomiting can be instituted in the following conditions.


A. TRUE Insecticide poisoning
B. TRUE Paracetamol poisoning
C. FALSE Childhood eating disorders
D. FALSE Poisoning with kerosene
E. FALSE Poisoning with acid or alkalis

45. Equipment and supplies for neonatal resuscitation include


A. TRUE Small ambu bag and suction apparatus
B. TRUE Oxygen cylinder syringes
C. FALSE Broad spectrum antibiotics
D. TRUE Warm clothes and intubation equipment
E. FALSE Big ambu bag and insulin.

46. The following are the signs present in a baby where meningitis is suspected
A. FALSE Paralysis of one or more limbs
B. TRUE Bulging of anterior fontanelle
C. FALSE Unequal dilated pupils
D. TRUE Positive Kerning’s signs
E. TRUE Neck stiffness

47. The following is found to be associated with down syndrome


A. TRUE Ventricular septal defects
B. FALSE Muscle hypertonia
C. TRUE Hyperthyroidism
D. TRUE Strabismus
E. TRUE A flat nasal bridge

48. Induction of vomiting can be instituted in a child poisoned with;


A. FALSE Kerosene
B. TRUE Insecticides
C. TRUE Paracentamol
D. FALSE Hydrochloric acid
E. FALSE Sodium hydroxide

49. Iron therapy should not be given to patients with the following condition
A. FALSE Heart failure
B. TRUE Kwashiorkor
C. FALSE Cerebral palsy
D. TRUE Signs of infections

CMT BANK OF QUESTIONS Page 42


REVIEW QUESTIONS PAEDIATRIC

E. TRUE Sickle cell disease

50. Requirement for the treatment of croup syndrome are


A. TRUE Maintain airway
B. TRUE Give nebulizer adrenaline
C. FALSE Start IV antibiotics immediately
D. TRUE Keep the child comfortable in semi-seated position
E. FALSE Isolate the baby to avoid transmission of the disease

51. The following are signs and symptoms of very severe pneumonia
A. FALSE Tachypnea
B. TRUE Central cyanosis
C. TRUE Not able to drink
D. FALSE Stridor when calm
E. TRUE Lower chest indrawing

52. The following are components of treatment of asthma;


A. TRUE Patient education
B. FALSE Intravenous antibiotics
C. FALSE Give oxygen 1litre/minutes
D. TRUE Avoidance or control of asthma triggers
E. TRUE Establishment of comprehensive plan of pharmacological therapy

53. Management of a patient with nephrotic syndrome


A. FALSE A 12-hours urine for protein
B. FALSE Blood tests for hepatitis A and D antibodies
C. TRUE Throat swab for culture to detect Group A streptococcus
D. TRUE Prednisolone and fluid resuscitation
E. FALSE Bed rest and high dose of frusemide

54. In patient with sickle cell disease


A. FALSE Acute chest syndrome is treated by oxygen therapy and steroid
B. FALSE Aplastic crisis presents with pallor, thrombocytosis and bradycardia
C. FALSE Prophylaxis with ferrous sulphate 5mg daily, and chloroquine 5mg/kg weekly
to prevent malaria
D. TRUE Osteomyelitis is one of the complications
E. TRUE Sickle cell crisis is treated by IV fluids, analgesics and IV antibiotics

55. Regarding rheumatic fever


A. TRUE Is an autoimmune disorder
B. TRUE Sickle cell anaemia is one of the differential diagnosis
C. FALSE It is common at all age groups with the peak at 25 years old
D. FALSE Arthralgia is one of the major modified Jones criteria for its diagnosis
E. TRUE Mitral valve is mostly affected

CMT BANK OF QUESTIONS Page 43


REVIEW QUESTIONS PAEDIATRIC

56. Hilary, a months old child was brought with severe pneumonia. The following are the
presenting features
A. TRUE Lower chest indrawing.
B. FALSE Bulging anterior frontanelle
C. TRUE Cyanosis
D. TRUE Head nodding
E. FALSE Neck stiffness

57. Regarding neonatal resuscitation at dispensary level


A. ……...... If the baby is not breathing well can be stimulated by IM stat vitamin K
B. ……….. Baby with APGAR score of 4-6 needs resuscitation
C. ……….. If baby cannot establish spontaneous breathing after 20 minutes stop doing
resuscitation as a baby has severe brain damage.
D. ……….. Insert endotracheal tube for ventilation
E. ……….. Give oxygen therapy if available

58. Regarding bacterial meningitis


A. ………. May presents with convulsions
B. ………. CSF shows increased glucose levels
C. ………. Positive kerning sign is one the physical findings
D. ………. Otitis media is the one of the risk factors
E. ………. IV Metronidazole is the drug of choice

59. Gastric lavage is indicated in the following conditions:


A. ………. A child with pulmonary TB
B. ………. Kerosene ingestion
C. ………. Iron poisoning
D. ………. A child with hypoglycemia
E. ………. Foreign body aspiration

60. Regarding feeding of an exposed baby to HIV positive mother:


A. ………. Mixed feeding is essential for the better health
B. ………. It is recommended to have exclusive breast feed for 6 months
C. ………. Give expressed breast feeding for 6months
D. ………. Replacement feeding if is available, affordable, accessible and sustainable
E. ………. Presence of an oral thrush is the contraindication to breast feed

61. Regarding cancrum oris


A. ……… It is synominous to cleft palate
B. ……… It is a polymicrobial infection colonizes when immune is compromised
C. ……… Malnutrition is one of its predisposing factors
D. ……… It is common in children below 12 months
E. ……… Reconstructive surgery can be beneficial in specialized centres

CMT BANK OF QUESTIONS Page 44


REVIEW QUESTIONS PAEDIATRIC

62. Concerning foreign body aspiration in children:


A. ……… It is not an emergency condition
B. ……… Aspiration of oil may induce inflammation
C. ……… A child may present with history of choking
D. ……… There could be positive history of aspiration
E. ……… One of the complications is tracheoesophageal fistula

63. With reference to burn injury in children:


A. ………. In burn grade III there is charred and damage of the dermis
B. ………. Parkland formula is used to calculate the amount of fluid per day
C. ………. In grade I there is superficial damage of the epithelium
D. ………. Grade III burn injury the dermis is reserved
E. ………. Full thickness burns are black or white appearance, usually dry and painless

64. Regarding sickle cell disease in children


A. ………. Sequestration or aplastic crises requires urgent blood transfusion.
B. ………. Acute vaso-oclusive crises requires urgent blood transfusion
C. ………. Chloroquine is used as prophylaxis for malaria
D. ………. Usually diagnosed at the age of about 8 weeks
E. ………. Sickling crisis can be precipitated by infection and dehydration

65. A 4-year-old child diagnosed of sickle cell 3 years ago on regular medications present with
difficulty in breathing associated with chest pain, cough and fever. On examination some
pallor, dyspneic, febrile and fine crackles. The treatment of this child includes
A. ……….. Oxygen
B. ……….. IV fluids
C. ……….. Ferrous sulphate
D. ……….. Blood transfusion
E. ……….. Parenteral antibiotics

66. The following are ways of decontaminating a child who has ingested poison
A. ……….. Administration of antidote
B. ……….. Activated charcoal
C. ……….. Inducing vomiting
D. ……….. Gastric lavage
E. ……….. IV fluids

67. A 4-year-old child presents with abdominal swelling for 2 months. On examination; alert,
afebrile, palpable mass in the right upper quadrants which is firm, immobile an non-tender.
What are the differential diagnoses?
A. ………. Nephroblastoma
B. ………. Hydronephrosis
C. ………. Hepatomegaly
D. ………. Neuroblastoma

CMT BANK OF QUESTIONS Page 45


REVIEW QUESTIONS PAEDIATRIC

E. ………. Splenomegaly

68. A premature baby girl born at a gestation age of 32 weeks may present with which of the
following signs?
A. ………. Intact sucking reflex
B. ………. Firm ear cartilage
C. ………. Enlarged clitoris
D. ………. Hypotonia
E. ………. Lanugo

69. Clinical presentation of organophosphate poisoning


A. Pupils dilatation.
B. Lacrimation.
C. Dry mouth.
D. Sweating.
E. Salivation.

70. You have been assigned to counsel a mother of a newborn at your health facility on proper
breast feeding. Which of the following will you advise the mother for good attachment during
breast feeding?
A. ……….. The chin should not touch the breast.
B. ……….. The lower lip should be turned inward.
C. ……….. The mouth should be wide open.
D. ……….. More areola should be visible above than below.
E. ……….. The lower lip should be turned outward.

71. Typical features of bilirubin encephalopathy includes:


A. ……….. Opisthotonous posture.
B. ……….. Hypotonia
C. ……….. Backward arching of the neck.
D. ……….. Tongue biting.
E. ……….. Respiratory distress.

72. Mrs. Yanguyenu delivered a baby with an APGAR score of 3-9. The cause of low APGAR
score of this baby in the first minute could be due to:
A. ……….. Severe congenital malformations.
B. ………. Reflex apnoea due to excessive suctioning.
C. ………. Fetal distress.
D. ………. Brain injury.
E. ………. Inadequate artificial respiration.

73. A 4-year-old child was brought at OPD with the complaints of wetting bed during sleep since
birth. The child normally sleeps at late; there is no history of diabetes. On examination was

CMT BANK OF QUESTIONS Page 46


REVIEW QUESTIONS PAEDIATRIC

alert, per abdomen there were normal findings, ENT the child has nasal polyps. The
management of this child includes;
A. ………. Fluid intake should be limited before bed time
B. ………. Should punish the child whenever she wets the bed
C. ………. The child should avoid before bed time
D. ………. Refer the child to otolaryngologist for nasal polyp removal
E. ………. Should be given diuretics before sleep

74. A 2-year-old child was brought to the hospital with the complaint of failure to thrive for the
past 6 months. He mother died the seven months ago with unknown cause. On examination
he was moderately pale, bilateral lower limb pitting edema, weight of age was 3-SD, the rest
of the examination was normal. Which of the following substances are used in management
of this child?
A. ………. IV furosemide
B. ………. IV prednisolone
C. ………. 75
D. ………. IV broad spectrum antibiotics
E. ………. IV ringer lactate

75. Acute watery diarrhea with no dehydration can be managed with which of the following home
fluid?
A. ………. Commercial juice
B. ………. Children syrup
C. ………. Rice water
D. ………. Coconut juice
E. ………. Soft drinks

76. A 10 years old boy child was playing in the garden, he accidentally cut his foot with hoe and
treated with local herbs at home. Six days later, he developed a sudden onset of convulsion.
On examination he was alert, opisthotomous posture, spastic jaws. The wound was septic with
pus discharge. The management of this child includes:
A. ………. IV ampicillin and metronidazole
B. ………. IV diazepam
C. ………. Feeding through NGT
D. ………. Tabs Paracetamol
E. ………. IV Artesunate

77. On assessment of the newborn the following clinical feature were noted, the baby was pink in
colour and there was swelling on the head, about 5 x 5cm, fluctuant with bruises. The rest of
the examination were normal. Which of the following correctly describes the management of
this child?
A. ………. Incise the swelling
B. ………. IV ampicillin
C. ………. Counsel the mother that swelling will subside

CMT BANK OF QUESTIONS Page 47


REVIEW QUESTIONS PAEDIATRIC

D. ………. Needle aspiration for confirmation


E. ………. Continue with exclusive breastfeeding

78. Failure to initiate or sustain respiratory effort by a neonate soon after delivery most likely is
due to;
A. ……….. Use morphine during labour and delivery
B. ……….. Neuromuscular disorders
C. ……….. Vitamin K deficiency
D. ……….. Neonatal hyperthermia
E. ……….. Cephalohematoma

79. A 10-year-old boy was brought to the hospital with complaints of nocturnal enuresis and
frequent thirsty for about 1 month. Her mother reported that the child has increased frequency
of food intake recently despite of weight loss. The most likely microvascular complications
to develop in this child include;
A. ………. Coronary artery disease
B. ………. Retinopathy
C. ………. Nephropathy
D. ………. Atherosclerosis
E. ………. Obesity

80. Complications of Acute post-streptococcal glomerulonephritis include;


A. ………. Hypertensive encephalopathy
B. ………. Hypokalemia
C. ………. Hypocalcemia
D. ………. Uremia
E. ………. Metabolic alkalosis

81. The following are clinical features of cerebral palsy


A. ………. Neonatal seizures
B. ………. Abnormal muscle tone
C. ………. Lack of primitive reflexes
D. ………. Behaviour abnormality like lack of interest and sleep disturbances
E. ………. Failure to fix gaze while feeding

82. When performing neonatal resuscitation;


A. ………. Inject vitamin K for the baby to breath well
B. ………. Squeeze ambu bag 30 times/minutes
C. ………. If the APGAR score is 1-3 use endotracheal tube for artificial ventilation
D. ………. Suck mouth and throat for more than 15 seconds at a time
E. ………. If the baby is not breathing spontaneously after 20 minutes stop resuscitation

83. When using drugs in a child with Bronchial asthma, the following facts should be noted;
A. ………. Propranolol may cause an acute attack

CMT BANK OF QUESTIONS Page 48


REVIEW QUESTIONS PAEDIATRIC

B. ………. Hydrocortisone work with 1 hour to relieve bronchospasm


C. ………. Salbutamol usually stops inflammation process
D. ………. Aspirin may be the cause of an attack in some patients
E. ………. Steroid should be given to all patients with asthma

84. Concerning pathological jaundice, the following statements are facts:


A. ………. Occurs within 24 hours after delivery
B. ………. Typically occurs on the second or third day of life
C. ………. Can be caused by blood group incompatibility
D. ………. Can be caused by increased breakdown of fetal erythrocytes
E. ………. Severe jaundice that continue after 7 to 14 days of life

MULTIPLE CHOICE MATCHING ITEM QUESTIONS


Match disease in column A with the corresponding sign in column B, by writing a letter from
column B in the space provided in column A: The sign can only be used once.
DISEASE SIGN
1. A Acute bronchial asthma A. Expiratory wheeze
2. G Acute rheumatic fever B. Inspiratory wheeze
3. C Nephritic fever C. Proteinuria
4. D Pneumonia D. Tachypnea
5. F Pleural effusion E. Trachea shift to the affected side
F. Trachea shift to the opposite side
G. Polyarthritis
H. Cyanosis

Match the disease in column A with the complication in column B, by writing a letter from
column B in the space provided in column A. The complications can be used only once.
DISEASE COMPLICATIONS
1. C Bronchial asthma A. Atelectasis
2. D Glomerulonephritis B. Autosplenecctomy
3. F Meningitis C. Bronchiectaxis
4. H Mumps D. Heart failure
5. B Sickle cell diseases E. Impaired liver function
F. Loss of hearing
G. Myocarditis
H. Oophoritis

Match the following diseases in Column A against the signs in Column B


COLUMN A COLUMN B
1. G Monoplegia A. Only arms are affected
2. E Diplegia B. Paresis of one side of the body
3. C Quadriplegia C. All limbs are affected equally

CMT BANK OF QUESTIONS Page 49


REVIEW QUESTIONS PAEDIATRIC

4. I Paraplegia D. Head injury


5. B Hemiplegia E. Legs affected more than arm
F. Only legs are affected
G. Motor paralysis of one limb
H. Paresis of feet and hand

Match the following diseases in Column A against the signs in Column B


COLUMN A COLUMN B
1. G Katayama fever A. Bowel and bladder dysfunction
2. E Neurocysticercosis B. Haemolysis
3. I Iron deficiency anaemia C. Staphylococci
4. A Acute flaccid paralysis D. Age
5. C Impetigo E. Taenia solium
F. Trauma
G. Schistosoma mansoni
H. Adenopathy
I. Kolonychia

Match the following complications from column B with their corresponding disease from
column A.
COLUMN A – Diseases COLUMN B - Complications
1. F Sickle cell disease A. Pneumothorax
2. C Osteomyelitis B. Inflammation of cartilage
3. H Cerebral palsy C. Pathological fracture
4. G Measles D. Hemothorax
5. A Asthma E. Stress fracture
F. Stroke
G. Blindness
H. Mental retardation

Match the following causative organism from column B with the disease they cause from
column A
COLUMN A – Diseases COLUMN B – Causative organism
1. C Urinary tract infection A. Rotavirus
2. E Tungiasis B. Epstein bar virus
3. G Blood diarrhoea C. E. coli
4. F Rheumatic fever D. Group B streptococcus
5. B Burkitts lymphoma E. Flea
F. Group A streptococcus
G. Shigella flexneri
H. Cytomegalovirus

Match the investigations in column B to disease in column A

CMT BANK OF QUESTIONS Page 50


REVIEW QUESTIONS PAEDIATRIC

COLUMN A COLUMN B
1. G Tuberculosis A. ASOT
2. E Neonatal sepsis B. Indian ink
3. H Sickle cell disease C. Chest X-ray
4. A Nephrotic syndrome D. Blood slides
5. B Cryptococcal meningitis E. Blood culture
F. Full blood count
G. Sputum for AFB
H. Hb – electrophoresis

Match the antidote in column B with position in column A


COLUMN A COLUMN B
1. G Iron A. Atropine
2. C Opoid B. Glucagon
3. F Benzodiazepines C. Naloxone
4. A Organophosphate D. Pyridoxine
5. H Calcium channel blockers E. Octreotide
F. Flumazenil
G. Deferoxamine
H. Calcium chloride

Match diseases in column A with their corresponding causative agents in column B


COLUMN A – Diseases COLUMN B – Causative agents
1. H Bronchiolitis A. Mycobacterium tuberculosis
2. A Tuberculosis adenitis B. Vibrio cholera
3. G Chicken-pox C. Cryptosporidium parvum
4. C/B Diarrhoea D. Trichuris trichiura
5. D Rectal prolapse E. Mycobacterium bovis
F. Shigella
G. Varicella zoster
H. Respiratory syncytial virus

Match the following diseases in column A against their corresponding complications in


column B
COLUMN A – Diseases COLUMN B - Complication
1. D Pneumonia A. Ascites
2. H Whooping cough B. Hydrocephalus
3. G Measles C. Anaemia
4. C Dysentery D. Pneumothorax
5. B Meningitis E. Liver abscess
F. Fever
G. Encephalitis
H. Epistaxis

CMT BANK OF QUESTIONS Page 51


REVIEW QUESTIONS PAEDIATRIC

Match the following diseases in Column A with their corresponding investigation of choice
in Column B

COLUMN A – Disease COLUMN B – Investigation

1. C Intestinal obstruction A. Abdominal ultrasound


2. H Meningitis B. Blood culture
3. B Neonatal sepsis C. Erect abdominal X-ray
4. E Haemolytic disease D. Urinalysis
5. F Neonatal Jaundice E. Direct anti-globulin test (DAT)
F. Serum bilirubin
G. Haemoglobin level
H. Cerebral spinal fluid analysis

Match the investigations in column B to disease in column A.


COLUMN A COLUMN B
1. D Sickle cell disease A. Full blood count
2. E Cryptococcal meningitis B. Chest X-ray
3. C Tuberculosis C. Sputum for AFB
4. G Nephrotic syndrome D. Hb – electrophoresis
5. F Neonatal sepsis E. Indian ink
F. Blood culture
G. ASOT
H. Blood slide

Match the antidote in column B with poison in column A


COLUMN A COLUMN B
1. C Opoids A. Flumazenil
2. D Iron B. Calcium chloride
3. E Organophosphate C. Naloxone
4. B Calcium channel blockers D. Deferoxamine
5. A Benzodiazepines E. Atropine
F. Glucagon
G. Pyridoxine
H. Octreotide

Match the paediatric condition in ‘column A’ with the corresponding complication in ‘column
B’ by writing the letter of the correct responses from ‘column B’ in the space provided in
‘column A’. Each response from column B is used only once.
COLUMN A (Paediatric condition) COLUMN B (Complication)
1. ………Birth asphyxia A. Kernicterus
2. ………Burn B. Cerebral oedema

CMT BANK OF QUESTIONS Page 52


REVIEW QUESTIONS PAEDIATRIC

3. ………Neonatal sepsis C. Empyema


4. ………Neonatal jaundice D. Septic shock
5. ………Pneumonia E. Cerebral palsy
F. Hypovolemic shock
G. Ketoacidosis
H. Diarrhoea

Match the treatment from column B with their corresponding condition in column A
COLUMN A – Condition COLUMN B – Treatment
1. …………Severe dehydration with no A. IV fluid (RL) 15mls/kg over 1 hours
malnutrition in a 6 months old child B. IV fluid (RL) 20mls/kg as rapid as
2. ………… Severe dehydration with possible
severe malnutrition in alert child C. IV 10% Dextrose 5mls/kg stat then
3. …………Shock with severe IV fluids
malnutrition in lethargic child D. IV Ringer Lactate 70mls/kg over 2
4. ………….Shock in child without 1/2 hours.
severe malnutrition E. IV Ringers Lactate 3mls/kg over 3
5. ………….Severe dehydration with hours
severe malnutrition in unconscious F. Rehydration solution for
child malnutrition 5mls/kg every 30
minutes
G. Rehydration solution for
malnutrition 20ml/kg over 1 hours

Match the IMCI classification from column B with the corresponding criteria in column A
COLUMN A-Criteria COLUMN B – IMCI Classification
1. …………Yellow palms and soles 10 A. Malaria
hours post-delivery B. Pneumonia
2. …………Sunken eyes and irritable C. Severe dehydration
child D. Severe malaria
3. ………….Skin pinch goes back E. Severe jaundice
slowly and lethargic F. Severe pneumonia
4. …………..Cough and fast breathing G. Some dehydration
in a calm child H. No dehydration
5. …………..Fever, prostration and
MRDT positive

Match the following poisons/drugs in column A against their specific antidotes in column B
COLUMN A – Poison/drug COLUMN B - Antidote
1. D Carbon monoxide A. Atropine sulphate
2. H Methanol B. Glucose
3. F Paracetamol C. Flumazenil
4. A Organophosphate D. Oxygen
5. C Diazepam E. Carbon test
F. N. Acetylcysteine
G. Charcoal

CMT BANK OF QUESTIONS Page 53


REVIEW QUESTIONS PAEDIATRIC

H. Ethanol

Match the opportunistic infections form Column B with their corresponding clinical features
in Column A by writing letter of the correct response in the space provided
COLUMN A – Clinical feature COLUMN B – Opportunistic infections
1. …... Finger clubbing A. Shingles
2. …... Persistent diarrhea B. Pneumocystis jirovecii pneumonia
3. …… Painful swallowing C. Popular pruritic eruption
4. …… Nodular skin lesions D. Lymphoid interstitial pneumonitis
5. …… Dermatome painful rashes E. Kaposi’s sarcoma
F. Esophageal candidiasis
G. Cryptosporidiosis
H. Ameoebiasis

Match the treatments from Column B with their corresponding conditions on Column A by
writing the letter of the correct responses in the table provided
COLUMN A – Conditions COLUMN B – Treatments
1. Kernicterus A. Kangaroo mother care
2. Cord granuloma B. Nil orally
3. Cephalohematoma C. Phototherapy
4. Severe birth asphyxia D. Reassurance and observation
5. Ophthalamia neonatorum E. Silver nitrate pencil
F. Surgery
G. Tertracycline eye ointment
H. Vitamin K

Match the treatments from Column B with their corresponding disease/conditions in Column
A by writing the letter of the correct responses in the table provided
COLUMN A – DISEASE/CONDITIONS COLUMN B - TREATMENTS
1. Haemolytic disease of newborn A. Physiotherapy
2. Pneumocystic jiroveci Pneumonia B. Injection ceftriaxone
3. Cerebral palsy C. Folic acid
4. Sickle cell disease D. Ferrous sulphate
5. Cord granuloma E. Exchange transfusion
F. Silver nitrate pencil
G. Cotrimoxazole
H. Cotrimazole

Match the interpretations from Column B with their corresponding physical findings in
Column A by writing the letter of the correct responses in the table provided
COLUMN A – PHYSICAL FINDINGS COLUMN B - INTERPRETATION
1. Stony dullness A. Abdominal mass
2. Unsynchronized radial and femoral B. Ascites
pulse C. Right side heart failure
3. Diminished bowel sounds D. Paralytic ileus
4. Shifting dullness E. Pleural effusion

CMT BANK OF QUESTIONS Page 54


REVIEW QUESTIONS PAEDIATRIC

5. Raised JVP F. Intestinal obstruction


G. Pneumothorax
H. Coarctation of aorta.

Match the clinical presentation from Column B with their corresponding conditions in Column
A by writing letter of the correct response in the table provided
COLUMN A – Conditions COLUMN B – Clinical presentations
1. Meningitis A. Black water fever
2. Impetigo B. Ballous on the skin
3. Cerebral palsy C. Macula on the skin
4. Acute glomerulonephritis D. Unequal pupil dilatation
5. Severe malaria E. Weak upper and lower limb
F. Owl’s eyes sign
G. Vesicle lesion on the skin
H. Increased blood pressure

Match the side effects from Column B their corresponding ART drugs in Column A by writing
the letter of the correct response in the table provided.
COLUMN A - ART COLUMN B – Side effects
1. Zidovudine A. Severe skin reaction
2. Tenofovir B. Renal failure
3. Lopenavir C. Jaundice
4. Efavirenz D. Night mare
5. Nevirapine E. Anemia
F. Bluish mucus membrane
G. Constipation
H. Vomiting

Match the clinical features from Column A with their corresponding diseases in Column Bby
writing the letter of the correct response in the table provided
COLUMN A – Diseases COLUMN B – Clinical features
1. Diabetes Mellitus A. Priapism
2. Epilepsy B. Vomiting
3. Sickle cell disease C. Haematuria
4. Hemolytic disease of newborn D. Haematemesis
5. Acute glomerulonephritis E. Polyuria
F. Jaundice
G. Diarrhea
H. Convulsion

Match the most likely complications from Column B with their corresponding opportunistic
infections in Column A by writing the letter of the correct responses in the table provided.
COLUMN A – Diseases COLUMN B – Complications
1. Pneumocystic pneumonia A. Brain damage
2. Oesophageal candidiasis B. Thoracic abscess
3. Cryptococcal meningitis C. Pneumothorax

CMT BANK OF QUESTIONS Page 55


REVIEW QUESTIONS PAEDIATRIC

4. Herpes zoster D. Malnutrition


5. Pott’s disease E. Empyema thoracic
F. Mediastinitis
G. Pathological fracture
H. Post-herpetic neuralgia

SHORT ANSWERS QUESTIONS


1. Mention five (5) contraindications of inducing vomiting in a poisoned child
 Depressed consciousness
 Convulsion
 Coma
 Poisons with petroleum products e.g. kerosene
 Poison with corrosive products e.g. acids and alkali

2. List five (5) differential diagnosis of neonatal sepsis


 Meningitis
 Severe pneumonia
 Respiratory distress syndrome
 Congenital viral or protozoan disease
 Birth asphyxia
 Meningitis

3. Outline five (5) important things to be done to ensure warmth of a newborn


 Maintain a warm chain for all newborn
 Have a warm room for birth
 Dry the newborn immediately after birth
 Replace the towel or cloths used to dry newborn with second dry towel/cloth/blanket
 Keep the newborn’s head, soles and palm covered
 Keep the newborn in close contact with his or her mother
 Skin-to-skin is the best as in the ‘kangaroo mother care’ position
 Encourage all mothers to breastfeed on demand from birth
 Delay bathing the newborn for at least 6 hours, and better until 24 hours
 Teach mothers how to keep the newborn warm, including keeping baby’s body
covered, how to check for hypothermia (cold feet or body) and how to re-warm

4. A 14 days old baby is brought to you because of poor feeding, vomiting, excessive cry. The
mother also reported history of convulsion. On examination the child is febrile (T=390C), No
jaundice, No cyanosis but has bulging anterior frontanelle.
(a) What is the most likely diagnosis?
 Meningitis

CMT BANK OF QUESTIONS Page 56


REVIEW QUESTIONS PAEDIATRIC

(b) List two (2) investigations will you order to confirm the above diagnosis
 Lumber puncture for CSF analysis
 Blood culture and sensitivity
(c) Mention two (2) complications of the above diagnosis
 Hearing loss
 Subdural effusion
 Hydrocephalus
 Brain abscess
 Paresis, ataxia
 Visual impairment

5. A 3-years old who was apparently well has developed an acute episode of fever for 3 days
associated with history of seizures. On examination the child is lethargic
(a) List two (2) conditions that should be considered as differential diagnoses of the above
symptoms
 Complicated malaria
 Meningitis
 Hypoglycemia
 Urinary tract infection
 Septicemia / Sepsis
 Severe malaria
(b) Mention three (3) important investigations that you will order
 Random blood glucose
 MRDT/BS
 Lumber puncture for CSF analysis
 Blood for culture and sensitivity

6. A 7 years old child presents with nausea, vomiting and abdominal pain. Mother also reports
that she noted loss of weight for the past two months. On examination the child is dehydrated
will kussmaul Hyperventilation
(a) What is the most likely diagnosis?
 Diabetes ketoacidosis (DKA)
(b) Outline four (4) important points in management of this child
 Perform ABCD resuscitation
 Correcting dehydration aggressively
 Correcting acidosis and reverse ketosis with insulin and bicarbonate
 Screening and treating infections
 Avoid complication of therapy
 Weighing the patient
 Reverting blood glucose to normal
 Identifications and treatment of precipitating events

CMT BANK OF QUESTIONS Page 57


REVIEW QUESTIONS PAEDIATRIC

7. A 6 – months old infant present with history of watery diarrhea for past three weeks. On
physical examination the infant is restless and irritable, has sunken eyes and his skin pinch
goes backs slowly:
(a) What is the most likely diagnosis?
 Chronic watery diarrhea with some dehydration
(b) Mention how are you going to manage this patient
 ORS 75mls/kg for 4 hours. Then assess and change plan accordingly
 Tab zinc 20mg once daily for 10/7 days
 Encourage breastfeeding frequently
 Treat other conditions accordingly

8. Mention five (5) advantages of breastfeeding to the baby


 It contains all necessary nutrients for the first 6 months
 It is easily digested by the baby
 Promotes bonding and love between mother and baby
 Prevents disease to infants due to the presence of antibodies and macrophages in
colostrum
 Always fresh, ready, available, no special preparation or equipment needed
 Reduce risk of allergies and intolerance
 Contribute to maturation of the GI – tract via Lactobacillus bifidus factors
 Clean, biologically safe and cheap
 Helps to establish proper suckling

9. Amina is 25 years old lady delivered a male term baby with birth weight 3.2kg. List five (5)
primitive reflexes you will examine to this newborn.
 Moro reflex
 Grasp reflex
 Sucking reflex
 Rooting reflex
 Stepping reflex
 Tonic neck reflex
 Parachute reflex
 Landau reflex

10. A 3 months old baby with a birth weight of 3.5kg was brought at the village dispensary, her
mother complaining the baby has difficult in feeding and recurrent chest infection since birth
which resolve with antibiotics. On examination the baby was conscious, afebrile, and
cyanotic, on cardiovascular examination a murmur was heard on auscultation, body weight of
3.0kg and RR 65b/m. Other physical findings were normal.
(a) What is the diagnosis?
 Cyanotic congenital heart disease
(b) What are differential diagnosis?

CMT BANK OF QUESTIONS Page 58


REVIEW QUESTIONS PAEDIATRIC

 Tetralogy of fallot
 Truncus arteriosus
 Tricuspid atresia
 Total anomalous pulmonary venous return
 Transposition of great arteries
(c) How will you manage this patient?
 Refer to hospital

11. A 2 months old baby, presented with history of high grade fever, high pitched cry and he was
unable to suck for 3 days, on examination the baby appears lethargic, febrile and bulging
anterior fontanelle.
(a) What is the most likely diagnosis?
 Bacterial meningitis
(b) List three (3) investigation to be ordered at hospital level.
 Lumber puncture for CSF analysis
 Culture and Sensitivity of CSF
 Full blood picture.

12. Grace is a 11 years old school child, she attended at the health center complaining of lower
abdominal pain, low grade fever, vomiting for past four days, she also noted that her urine
colour was changed:
(a) What is the most likely diagnosis?
 Urinary tract infections
(b) Mention two investigations that you will order at health center
 Urinalysis
 MRDT/BS for MPS
(c) Mention the drug of choice for the above diagnosis
 Amoxicillin 250mg 8hourly for 5 days/ Contrimoxazole 480mg bid for 10 days

13. A 4-year-old child was brought to the dispensary by his mother complaining of that her baby
has productive cough for more than 2 weeks and low grade fever, she recently noted that her
baby has lost more than 1kg for the past one month; she reported that the baby has used several
antibiotics without improvement. On examination the baby is wasted with enlarged cervical
lymphnodes.
(a) What is your diagnosis?
 Pulmonary tuberculosis
(b) List two (2) investigations which you are going to perform at dispensary level
 Sputum for AFB
 Rapid test for HIV
 Chest X-ray
(c) Mention the plan of treatment of this child.
 Ant Tb – RHZE for two months
 RH – for four (4) months

CMT BANK OF QUESTIONS Page 59


REVIEW QUESTIONS PAEDIATRIC

14. You are a clinician at Mkomaindo dispensary and a 5-year-old child is brought to you with a
burn wound, briefly explain four important things to consider during clinical assessment
 Is the child in circulatory shock? (Thread pulse and cold skin on the extremities not
burn)
 Is the child breathing easily? (Internal burns /Inhalation or trauma)
 Is the child in pain?
 What is the body weight?
 How deep is the burn?
- Full thickness burns are black or white, usually dry, have no feeling and do not
blanch on pressure.
- Partial thickness burns are pink or red, blistering or weeping and painful.
 How much of the body is burned?
- Use a body surface area chart according to age
- Alternatively, use child’s palm to estimate burn area
- A child’s palm is approximately 1% of the body surface area.

15. Enumerate four (4) pre referral management for a child with haemorrhagic disease of newborn
at the health center.
- Haemoglobin level, blood grouping and cross matching.
- Stop visible bleeding (e.g. umbilical stump) with direct pressure and bandaging
- I.M. Vitamin K1 1 to 2mg stat.
- Blood transfusion with fresh blood.

16. Mention six (6) conditions specific to HIV infection in children


 Pneumocystic Jirovecii Pneumonia (PCP)
 Oesophageal candidiasis
 Cryptococcal meningitis
 Herpes zoster
 Kaposi’s sarcoma
 Lymphoma
 Progressive multi-focal encephalopathy
 Lymphoid interstitial pneumonitis

17. Kitwana, a 4-year old is brought to your dispensary having fallen from three and sustained
injury of the neck. Outline how you are going to manage this boy
 Assess airway, breathing and circulation
 Stabilize the child’s neck and keep the child lying on the back
 Tape the child’s forehead and chin to the sides of a firm board to secure his position.
 Prevent the neck from moving by supporting the child’s head
 If vomiting, turn on his side, keep the head in line with the body

CMT BANK OF QUESTIONS Page 60


REVIEW QUESTIONS PAEDIATRIC

18. You are a clinical officer on duty at Kasanga health centre. You are about to discharge a
mother who is 24 hours post-delivery. Outline how you are going to counsel her regarding
cord-care
 Wash hands before and after cord care
 Put nothing on the umbilical stump
 Fold nappy (diaper) below the umbilical cord stump.
 Keep the umbilical stump loosely covered with clean clothes
 Seek care if the umbilical or surrounding skin is red or draining pus or blood
 Do not bandage the umbilical stump
 Avoid touching the umbilical stump unnecessary.

19. A 3-days-old neonate born at gestation age of 32 weeks in brought to a dispensary with a
history of sudden onset of difficulty in breathing. Physical examination reveals it is dyspnoeic,
respiratory rate 66 breaths per minutes and has chest in drawing
(a) What is the most likely diagnosis?
 Acute Respiratory distress syndrome

(b) What might be the underlying causes of the problem?


 Prematurity
 Surfactant deficiency

(c) Outline the treatment of this neonate


 Clear the airways and give oxygen if available
 Keep warm
 Give broad spectrum antibiotics such as Ampicillin IV
 Expressed breast milk by cup and spoon via NGT
 Refer to hospital

20. With regard to current immunization schedule, outline five (5) vaccine and site of
administration that are provided to children before nine (9) months of age:
 OPV - Mouth
 BCG - Right shoulder
 DPT – HB - Left thigh
 PCV 13 - Right thigh
 Rotavirus - Mouth

21. Mention five (5) equipment/supplies used for resuscitation of the newborn baby
 Ambubag
 Suction apparatus
 Oxygen
 Intubation equipment
 Syringes

CMT BANK OF QUESTIONS Page 61


REVIEW QUESTIONS PAEDIATRIC

 Resuscitation table
 Warm clothes (Blanket, linen)

22. A 6 years old child was brought to health facility with complaint of poison ingestion. Mention
five (5) contraindication of inducing vomiting to this child.
 Depressed consciousness
 Convulsions
 Comma
 Poisons with petroleum products
 Poison with corrosive products, acids and alkalis

23. A newborn present to you with history of difficulty in breathing. On examination, a newborn
was dypnoec with respiratory rate of 78 breaths per minutes and lower chest indrawing
(a) What is most likely diagnosis?
 Respiratory distress syndrome.
(b) Mention four (4) causes of the above diagnosis
 Meconium aspiration pneumonia
 Septicemia
 Pulmonary haemorrhage
 Amniotic fluid aspiration
 Congenital heart diseases
 Identical herpes simplex

24. (i) What are the first recommended first line antiretroviral (ARV) drug regimens for the
children under 3 years old?
 AZT + 3TC + NVP
 ABC + 3TC + NVP
(ii) Which criteria you will consider before initiating antiretroviral therapy in an infant below
18 months of age:
 All HIV infected below 12 months of age with biologically proven infection
irrespective of WHO clinical staging and CD4 percent.
 HIV exposed infant under age of 12 months
 HIV exposed infant aged 12 – 18 months in WHO paediatric stg. 1 or 2 with CD4
less than 20%.
 Should initiate ART in HIV exposed infant aged below 18 months in WHO
paediatric stage 3 or 4 without virological confirmation or CD4 percent.

25. A 10 months old child weighing 9kg presents at your clinic with a history of diarrhea for 8
days. There is no blood in stool. Upon examination, the child is alert, has sunken eyes, drinks
eagerly and the skin pinch goes back very slowly
(a) Diagnosis
 Acute watery diarrhea with severe dehydration
(b) Treatment

CMT BANK OF QUESTIONS Page 62


REVIEW QUESTIONS PAEDIATRIC

 IV Ringer’s Lactate or Normal Saline 100ml/kg for 6hours as follows


- 30mls x 9kg = 270mls in the first 1 hours
- Then 70mls x 9kg = 630mls in the next 5 hours, then assess and classify
dehydration status after finishing the fluid
 Give paediatric zinc 20mg PO OD for 10-14/7 days
 Encourage breastfeeding.

26. List two general principles of administering oxygen


 Give right indication
 Right concentration
 Oxygen should be considered as a drug

27. List down two (2) methods of oxygen administration in paediatrics


 Low flow oxygen through nasal catheter
 Oxygen by mask
 Oxygen into a head box

28. Andrew ages 18 months, is brought to the dispensary by his mother who reports that the child
has feeding difficulties, inability to suck and had rapid jerky movement mainly proximal parts
of arms and legs. You conduct a physical examination and discover that Andrew has signs of
cerebral palsy. How will you manage Andrew’s condition at your health center?
 Parental counselling
 Support and encouragement of the parents
 Physiotherapy
 Education of the family and community
 Refer to the hospital for further management

29. Outline five (5) symptoms included in IMCI quick criteria for diagnosis of HIV infection:
 Recurrent pneumonia
 Oral thrush
 Persistent diarrhea
 Very low weight
 Enlarged lymphnodes
 Parotid enlargement
 Severe malnutrition
 Otitis media

30. Outline the management of Acute flaccid paralysis at a Health centres


 Bed rest
 Avoid injections
 Take stool for virus isolation (send to reference laboratory)
 Prevent contracture by splinting limbs

CMT BANK OF QUESTIONS Page 63


REVIEW QUESTIONS PAEDIATRIC

 Prevent bed sores

31. A one-day old infant is brought to the clinic. She is oozing blood from umbilical stump since
3 hours ago. Explain the management of this baby at health center
 Stop visible bleeding
 Check the cord ligature
 Vit K1, 1-2mg IM stat
 Check HB level
 Refer to hospital

32. Outline the management of a child who has ingested kerosene. His breath smells of kerosene
 Do not induce vomiting
 Do not attempt gastric lavage
 Give milk of magnesia
 Antibiotics
 Admit and observe

33. Enumerate five features of diabetic ketoacidosis in children


 Loss weight
 Dehydration
 Nausea and vomiting
 Kusmaul’s breathing
 Sweeting smelling breath (Acetone smell)
 Impaired sensation
 Altered levels of consciousness

34. Identify risks of early cessation of breast feeding:


 Malnutrition
 Dehydration
 Increased risk of infections
 Engorgement of mothers breasts
 Mastitis of mother
 Depression of mother
 Increased risk of pregnancy to mother

35. A neonate presents at health center with convulsions. Outline the management of this infants:
 Make sure the airway is clear and neonate is breathing
 Stop convulsion with phenobarbitone 10-20mg/kg IV
 Prevent and treat hypoglycaemia – Dextrose 10% 5ml/kg IV
 Expressed breast milk – by cup and spoon/pass NGT
 Pre-referral antibiotics – Inj. Chloramphenical IM
 Refer to hospital

CMT BANK OF QUESTIONS Page 64


REVIEW QUESTIONS PAEDIATRIC

36. Mention five diagnostic criteria for childhood tuberculosis


 Cough for more than 2 weeks
 Failure to strive or weight loss
 History of TB contact
 Chronic malnutrition, not improved after 4 weeks
 Chronic infant diseases not responding to antibiotics
 Suggestive X-ray-cavitation or enlarged lymph node
 Cervical/ mandible lymphadenopathy
 Swelling of bone or joint
 Angle joint deformity

37. Mention five goals of antiretroviral therapy in children:


 Promote survival
 Promote growth and development
 Reduce chances of opportunistic infections
 Suppress HIV infection
 Improve quality of life/Reduce chances of illness

38. List down four (4) most relevant investigations to order in a child suspected of nephrotic
syndrome
 Urine for protein (24 hours or urine analysis)
 Plasma albumin and cholesterol
 Renal function test
 FBP + ESR
 Serum ASO titre (ASOT)
 Throat swab for culture

39. Briefly explain five (5) local features used in paediatric score chart for diagnosis of TB
 Lymph node – Cervical, submandibular – score 3
 Chest x-ray – TB suggestive feature like infiltration, cavity or hilar lymphnode
 Swelling of bones and joints – Suggestive features on X-rays
 Ascites – Without abdominal mass, with abdominal mass
 Meningitis – Chronic CNS signs
 Angle deformity of the spine – X-ray feature

40. Toto is an 8years old boy presenting with fever, joint pain on the left knee later involves the
right ankle joint for 3 days. On examination he was febrile (T=380C), cardiovascular system
examination a mid-diastolic murmur was heard on mitral area. Other physical findings were
normal
(a) What is the most likely diagnosis?
 Rheumatic fever
(b) What are the criteria you have used to arrive to the above diagnosis?

CMT BANK OF QUESTIONS Page 65


REVIEW QUESTIONS PAEDIATRIC

 Major criteria: Migratory polyarthritis and carditis


 Minor criteria: fever
(c) What is the risk factor for the above diagnosis?
 Group A streptococcal sore throat infection

41. During inspection of chest in the respiratory system examination in child, what are the five
(5) important thing you should look for?
 Respiratory rate
 Nasal flaring
 Breast, nipples, any deformity, marks, scar and visible pulsation
 Look for any asymmetry of the chest
 Precordial bulging, hyperactivity

42. A 10 years old girl present with severe pain on the upper and lower limbs and yellowish
colouration of the eyes, no history of fever, her mother reports history of several blood
transfusions to her daughter, on examination she was afebrile T=370C and jaundiced.
(a) What is the most likely diagnosis?
 Sickle cell disease – Painful vasooclusive crisis
(b) Mention one investigation at the hospital to confirm the above diagnosis
 Sickling test
 Hb electrophoresis
(c) List six (6) complication of the above diagnosis
 A vascular necrosis of the head of femur
 Osteomyelitis
 Chronic ulcer
 Priapism
 Stroke
 Blindness
 Gall stones
 Autoplenectomy

43. With examples outline five (5) causes of anaemia


 Nutritional deficiency, examples - iron, vitamin B12 and folate deficiency
 Impaired absorption, examples - persistent diarrhea, impaired absorption
(malabsorption) , intrinsic factor deficiency
 Chronic diasease, examples – renal, malignancy, infection –TB
 Infection/infestion, examples – malaria and hookworm
 Bone marrow failure, examples – drugs, radiation, malignancy

44. A 16months old male child, brought to the dispensary by his mother who reports that child
has feeding difficulties, inability to suck, unable to stand and has rapid jerky movement,

CMT BANK OF QUESTIONS Page 66


REVIEW QUESTIONS PAEDIATRIC

mainly proximal parts of arms and legs. On physical examination you find he has increased
muscle tone on both upper and lower limbs
(a) Diagnosis
 Cerebral palsy
(b) Perinatal causes
 Birth asphyxia
 Intracranial bleeding
 Abruption placentae
 Jaundice and infection
 Anoxia caused placenta previa

45. A 5 years old boy, known sickler presents at your clinic with complaints of generalized pain
in the limbs and joints, difficulty in breathing and fast breathing for two days.
(a) Identify the crises that child is experiencing
 Painful crisis
 Acute chest syndrome

(b) Outline how important things to be included in his management


 Do haemoglobin estimation
 Give intravenous fluids (Normal saline or Ringer’s lactate)
 Give analgesics
 Give antibiotics to treat infections
 Refer to hospital

46. Megaloblastic anaemia


(a) Other name is Macrocyclic anaemia
(b) RBC’s appears Large
(c) What are the causes
 Folic acid or Vitamin B12 deficiency.
 Poor absorption or poor utilization of its nutrients
 Increased loss of nutrients

47. (a) Hypersplenism is a clinical syndrome in which cytopenia results when the splenic function
become excessive as it enlarges
(b) This has been attributed due to the following possible mechanism;
 Excessive splenic phagocytic function
 Splenic production of antibodies that results in destruction of hematopoietic cells
 Over activity of splenic function
 Splenic sequestration

48. John a 10 months old baby was brought to OPD with history of itchy skin rashes on the cheeks
and flexures of neck and elbows. This started at the age of 3 months and is progressing with

CMT BANK OF QUESTIONS Page 67


REVIEW QUESTIONS PAEDIATRIC

time. These started as reddish papules now they have developed into vesicles and area of
hyperpigmentation
(a) What is the diagnosis?
 Infantile eczema
(b) What is the causes?
 Atopy
 Genetic factor
 Hypersensitivity/ Allergy
(c) How will you manage this patient?
 Topical steroids e.g. Hydrocortisone cream
 Antihistamine e.g. Promethazine
 Antibiotics if there is secondary bacterial infections

49. Juma aged 2 years is brought to the hospital by his mother who reports that the child has
feeding difficulties, inability to suck and has rapid jerk movements, mainly of proximal parts
of arms and legs. On physical examination you discover that Juma has signs of cerebral palsy.
How would you manage this patient?
 Parental counselling
 Physiotherapy
 Speech therapy
 Anticonvulsants
 Regular clinic attendance

50. Diana a 5 years old Masai child was brought to OPD with history of being lethargic and weak
for the past 2 weeks. She like to sleep most of time. She has no fever. On examination she
looks small for her age, body weight 12kg, and height of 92 cm which lie on the -2SD of the
WHO chart. She also had severe pallor of her palms and mucus membranes. Her vitals:
temperature 36.6 degree Celsius, PR 100/min, RR 43/min.
(a) What are the diagnoses?
 Severe anaemia
 Malnutrition (Underweight – Wt/Ht -SD
(b) What investigation will you do?
 Full blood picture or Haemoglobin level estimation
 Stool analysis
 MRDT
 Blood grouping and cross matching
 HIV test

51. Mention 5 cyanotic congenital heart diseases


 Teratology of Fallot
 Transposition of the great arteries
 Tricuspid atresia

CMT BANK OF QUESTIONS Page 68


REVIEW QUESTIONS PAEDIATRIC

 Total anomalous pulmonary venous return


-

52. Mention five (5) acyanotic congenital heart diseases


 Atrial septal defects
 Ventricular septal defects
 Patent ductus arteriosus
 Coarctartions of aorta
 Atrio ventricular canal defects

53. What are the causes of sensory neural hearing loss?


 Intrauterine infections e.g. Rubella
 Acquired infection e.g. Meningitis
 Perinatal complications e.g, Birth asphyxia
 Ototoxic drugs e.g. Aminoglycosides
 Tumors and drugs given for their treatments.

54. Chausiku was diagnosed to be HIV positive at the first antenatal visit with gestation age of 28
weeks and kept on ART. Soon after delivery she was anxious to breast fed baby as the baby
may get infected with HIV. Outline five breastfeeding options you will advise Chausiku to
minimize possibility of infecting her baby.
 Advice on exclusive breast feeding from birth to six months
 After six months she can stop breast milk and give other milk and food
 Mother should breastfeed on demand day and night
 Ensure correct positioning and attachment
 Give no bottle, teats and dummies
 Treat oral candidiasis and other oral lesion in the baby
 Treat vaginal/oral candidiasis in mother

55. You are a clinician at mikanjuni health center and mother deliver a new born with APGAR
score of five (5). Briefly explain how will you manage this newborn at your facility.
 Clear airways by suction of wiping
 Ventilate the baby by using ambubag or oxygen if available
 Positioning
 Keep stimulate new born with drying and rubbing back and extremities
 If convulsing give phenobarbitone 10-20mg/kg IM
 Refer to hospital

56. A newborn baby obtained an APGAR score of 4 soon after delivery at the dispensary. Outline
five (5) important management plan for this patient

CMT BANK OF QUESTIONS Page 69


REVIEW QUESTIONS PAEDIATRIC

57. A newborn present to you with history of difficulty in breathing. On examination, a newborn
was dyspneic with respiratory rate of 78 breaths per minutes and lower chest in drawing.
(a) What is the most likely diagnosis?
 Respiratory distress syndrome
(b) Mention four (4) causes of the above diagnosis
 Meconium aspiration syndrome
 Septicemia
 Pulmonary haemorrhage
 Amniotic fluid aspiration
 Congenital heart disease
 Identical herpes simplex.

58. A 6 years old child was brought to the health facility with complaint of poison ingestion.
Mention five (5) contraindication of inducing vomiting to this child.
 Depressed consciousness
 Convulsions
 Comma
 Poisons with petroleum products
 Poison with corrosive products like acids and alkali

59. With regard to current immunization schedule, outline five (5) vaccine and site of
administration that are provided to children before nine (9) months of age.
Vaccine Site of administration
OPV Mouth
BCG Right shoulder
DPT – HB Left thigh
PCV 13 Right thigh
Rotavirus Mouth

60. Mention five (5) differential diagnoses for a child of two years who present with fever and
loss of consciousness.
 Severe malaria
 Meningitis
 Septicemia
 Viral encephalitis
 Typhoid fever

61. Nelson is 3 years old child. He was brought at Health center due to the following complaints;
abdominal pain with progressive abdominal distension, in ability to pass stool and flatus and
vomiting two days
(a) What is the most likely diagnosis?
 Intestinal obstruction

CMT BANK OF QUESTIONS Page 70


REVIEW QUESTIONS PAEDIATRIC

(b) Mention four (4) pre referral management to be given to this child
 IV fluids – Ringer’s lactate and Normal saline
 Parenteral antibiotics – Ampicillin / Chloramphenicol
 Nasogastric tube for decompression
 Urethra cauterization

62. Mention five (5) equipment/supplies used for resuscitation of a newborn baby.
 Ambubag
 Suction apparatus
 Oxygen
 Intubation equipment
 Syringes
 Resuscitation table
 Warm clothes (blanket, linen)

63. A 3-year-old-boy, who weighs 14kg, brought to the health center 9 hours after sustaining burn
injury. On examination, the burn was on the trunk and involve about 25% total body surface
area. Outline five (5) management plans for this child

64. Mention five (5) clinical features that may suggest meningitis in a 4-month-old baby
presenting with hyperpyrexia

65. A 3-hour-old baby developed difficulty in breathing associated with grunting. On examination
temperature was 36.80C with respiratory rate of 76 breaths per minute.
(a) What is the most likely diagnosis?
(b) Outline four (4) management of this patient

66. Outline five (5) precautions when caring the cord of a newborn.

67. Outline four (4) contraindication of lumber puncture.

68. Enumerate four (4) typical feature of diabetic ketoacidosis.

69. Outline four (4) equipment and supplies for resuscitation.

70. Outline four (4) differential diagnoses of a 8-months-old child with convulsion.

71. A 9-year-old girl presents to the hospital with history of difficulty in breathing, ease
fatigability and awareness of heartbeat. On examination: Severe palmar pallor, Temp 36.40C,
bilateral lower limb pitting edema. Outline four investigations which will establish the cause
of her condition.

CMT BANK OF QUESTIONS Page 71


REVIEW QUESTIONS PAEDIATRIC

72. A 10-month-old child who is severely malnourished is brought to the dispensary after having
single episode of convulsion 1 hour prior. On examination: lethargic and the neck is soft.
Investigations reveal MRDT – negative and Random Blood Glucose (RBG) is 1.5 mmol/L.
(a) What could be the most likely cause of convulsion?
(b) Outline the four (4) important managements to this condition

73. Breastfeeding is important to the newborn in much aspect. All mother of newborn should be
counseled on good breastfeeding practices. Concerning this
(a) List two (2) signs of readiness of the baby to breastfeed
(b) Outline three (3) signs of good attachment

74. A 5-year-old boy was brought to the dispensary with the history pf sudden onset of severe
abdominal pain, vomiting and failure to pass stool. On examination he was alert, afebrile and
not pale. The abdomen is distended with exaggerated bowel sounds.
(a) What is the most likely diagnosis?
(b) Outline the management plan at that health facility

75. Mussa, a newly employed clinical officer at Mwanzugi dispensary, observed a long waiting
queue at his consultation room of mothers who brought their sick children for treatment. List
down any five (5) priority signs which he needed to assess in order to triage those sick children

76. Mention five (5) causes of respiratory distress in neonates


77. List down five (5) predisposing factors of Cancrum Oris (neonatal jaundice).
78. A preterm newborn was brought to dispensary with the history of oozing blood from the
umbilical cord 24 hours after delivery. During assessment, she was pale and active bleeding
from the cord was noted. Bed side bleeding time was prolonged
(a) What is the mostly likely diagnosis?
(b) Outline the management of this newborn at that health facility. (Four points)
79. Mention five (5) components of essential newborn care.

80. You have a plan to deliver health education on breastfeeding to postnatal mothers. Outline
five (5) importance of breast feeding to the mother which you will need to educate those
mothers

81. A 2-year-old boy presents with delayed milestone and feeding difficulties. On examination;
microcephaly, Head lag and drooling saliva and stiffness of both upper and lower limbs. Other
physical findings are normal.
(a) What is the most likely diagnosis?
(b) List four (4) complications

82. A clinician has been called to review a newborn who presented with difficulty in breathing
immediately after birth. He was born prematurely at gestation age of 32 weeks. On
examination : Alert, dyspneic, afebrile, central cyanosis and lower chest wall indrawing.
(a) What is the most likely diagnosis?

CMT BANK OF QUESTIONS Page 72


REVIEW QUESTIONS PAEDIATRIC

(b) List four (4) differential diagnosis

83. A newborn present with inability to cry soon after delivery. On examination: limp, gasping,
cyanosis, pulse rate is 90 b.p.m and poor reflexes
(a) What is the most likely diagnosis?
(b) List down four (4) equipment for resuscitation of this child

84. A clinician is performing clinical assessment of a 3-year-old girl who sustained burn this
morning. Outline five (5) parameters he will assess to this child

85. Outline five (5) perinatal causes of mental retardation

86. A 1-day-old baby presents with bleeding at the injection site after being vaccinated for BCG.
He was born premature at gestation age of 33 weeks. On examination: alert, afebrile, pallor,
active bleeding at the site of injection and cold extremities
(a) What is the most likely diagnosis?
(b) Outline the management of this child at dispensary level.

87. A 5-year-old boy, known asthmatic presents to the health center with difficulty in breathing
for 3 days which has been progressively increasing in severity. On examination: restless,
afebrile, sweating, tachycardia, cyanosis, tachypnea, lower chest wall in drawing and audible
wheezing. Mention five (5) factors which could have precipitated the condition

88. A 3-year-old girl was found unconscious in a garden with an open pesticide bottle beside her.
On examination: unconscious, afebrile, excessive secretions in the mouth and constricted
pupil and clothes are soaked with pesticides

(a) What is the most likely diagnosis?


(b) Outline the treatment of this child at dispensary level (4pts)

89. A 2 years old boy was brought to a dispensary with high grade fever, while waiting in the
queue he developed tonic clonic convulsions with tongue biting. Outline the immediate
management for this child: (five points).

90. A 10-year-old child known sickle cell patient on regular medication came at the dispensary
with complain of difficulty in breathing for 6 hours. On examination severe pale with tinge
jaundice. Per abdominal examination noted asymmetrical abdominal distension, tender in the
left hypochondriac region with splenomegaly. There was delayed capillary refill. Other
systems were normal.
(a) What is the most likely diagnosis?
(b) Outline four (4) pre-referral management plans for this patient.

CMT BANK OF QUESTIONS Page 73


REVIEW QUESTIONS PAEDIATRIC

91. A newborn baby delivered at the dispensary within 24 hours. The child developed yellowish
discoloration of the body. On examination; had jaundice on the trunk and the sores, the rest of
the examination was normal. T=38.50C. Outline five (5) management plans for this baby.

92. A 9-year-old child presented at the dispensary with complains of poor performance in school.
She had uncontrollable movement and there was facial grimacing which is exacerbated by
stress and disappears with sleep for 6 months. On examination the child has macular lesion
with pale center, not pruritic, distributed on the trunk and also subcutaneous nodule noted.
(a) What is the provisional diagnosis?
(b) Outline four (4) treatment plans for this child.

93. List five (5) Jones major criteria for diagnosis of Rheumatic fever

94. Outline five (5) goals of antiretroviral therapy for HIV positive children.

95. Outline five (5) types of crises observed in sickle cell disease.

96. A 3-year-old child was brought to the dispensary with complaints if high grade fever and
altered level of consciousness for 1 day. No history of discomfort during micturition, diarrhea
or vomiting. On examination; lethargic, no cyanosis, some palmar pallor with delayed
capillary refill. Vitals; T=39.50C, PR=138bpm, regular with weak volume, RR=38bpm.
MRDT is negative. Outline five (5) pre-referral management of this child.

97. A 3-day-old child was brought to the hospital with complaints of inability to suck and fever
for days. On examination; lethargic, some pallor, scleral jaundice, febrile with poor suckling
reflex. MRDT is negative.
(a) What is the most likely diagnosis?
(b) Outline the four (4) treatment plans for this child

98. A 2-day-old girl was brought to the OPD due to irritability, sleeping disturbance. Her mother
reports that the child is unable to sit since birth. On examination: is unable to fix gaze while
feeding with muscle stiffness.
(a) What is the most likely diagnosis?
(b) Outline four (4) complications of above diagnosis.

99. Outline five (5) points which must be taken into consideration when counselling on Kangaroo
mother care according to KMC guideline.

100.Outline five (5) signs of good attachment during breastfeeding which a health care provider
should explain to the mother in Kangaroo mother care (KMC)

101.Outline five (5) criteria for discharging Kangaroo Mother Care

102.Outline five (5) Contraindications for inducing vomiting in a poisoned child.

CMT BANK OF QUESTIONS Page 74


REVIEW QUESTIONS PAEDIATRIC

103.A newborn presented at the dispensary with failure to establish spontaneous breathing soon
after delivery. On examination; the newborn was lethargic, bluish discoloration of the skin
and RR is breath per minute, RBG IS 4mmol/l
(a) What is the most likely diagnosis?
(b) Outline four (4) pre-referral treatment plan for this baby

104.A 2-year-old child is brought to the dispensary with complaints of passing loose stool for 2
days. The stool is watery and had about three motions per day. On examination; the child
drinks normally and skin turgor goes back normally.
(a) Classify this child according to IMCI guideline
(b) Outline four (4) treatment rules to counsel this child’s mother

105.A 4-year-old boy was brought to the dispensary after sustaining burn injury to the anterior
trunk and right upper limb. On examination; alert, not pale, not dyspneic. Blisters involving
the burn areas noted
(a) List two (2) early complications for this child
(b) Mention three (3) late/long term complications for this child

106.Diagnosis of Tuberculosis is challenging sometimes in children presenting with cough and


difficulty in breathing especially when bacteriological confirmation is absent. This lead to the
development of TB score to aid in making diagnosis. Considering this; outline five (5) general
features which may be used for diagnosis of Tuberculosis using TB score.

107.The normal respiratory rate of an LBW and preterm infant ranges between 30 and 60 breaths
per minutes and breathing alternate with intervals of no breathing (apnoea) in kangaroo mother
care (KMC). Outline five (5) important measures of engaging the mother in monitoring apnea
at KMC premises.

108.A 4-year-old was brought to the health center with history of fever. This is the first time to be
brought to the health facility because of illness since birth. After diagnosis, a clinician decided
to give intravenous ampicillin. During administration of the drug, the child developed
difficulty breathing, Temperature noted to rise by 30C from the baseline and PR=112bpm.
(a) What is the most likely diagnosis?
(b) Outline four (4) immediate things the clinicians should do.

109.An 8-year-old present to the hospital with complaints of right ear pain and mild fever fir 2
weeks. There is history of hearing loss without any ear discharge noted before on the same
ear. On examination: swollen, tender on the right post-auricular area with erythematous
overlying skin. The right pinna is displaced outward and upward. There is mild hearing loss
in the same ear.
(a) What is the most likely diagnosis?
(b) Outline four (4) complications of this child’s condition

CMT BANK OF QUESTIONS Page 75


REVIEW QUESTIONS PAEDIATRIC

ESSAY QUESTIONS
1. Explain the components of essential newborn care
Definitions
 All essential activities done to ensure wellbeing and proper growth of the neonates

Ensure warmth for the baby


 Dry newborn completely immediately after birth
 Replace the towel used to dry the baby with another dry towel, cloth or blanket
 Provision of warm clothes after birth
 Use kangaroo mother care frequently
 Continue to breastfeed the child on demand from birth

Breastfeeding
 Put the newborn together with her mother in the first hour after birth.
 Help the mother with first breastfeeding
 Kangaroo mother care need to be encouraged
 Breastfeeding must continue

Cord care
 Wash hands before and after cord care
 Put nothing on umbilical cord stamp
 Keep umbilical cord stump loosely covered with clean clothes
 Explain to mother that she should seek care if umbilicus our surrounding skin is red
draining pus or blood
 Do not bandage umbilical cord stump on abdomen
 Avoid touching umbilical cord

Eye care
 Clean baby’s eyes immediately after birth by swabbing each eye separately with a
clean cloth dipped in NS or boiled or cooled water
 Apply tetracycline eye ointment after cleaning and with one hour after birth
 Do not touch eye with the tip of ointment tube
 All wash hands before and after taking care of the baby

Routine immunization
 Counsel mother on the importance of immunizing her baby
 Advise mother that even small baby can be immunized without harm
 Immunize baby with BCG vaccine for TB and first dose of OPV
 Explain the side effects which may occurs and how to treat them
 Emphasize the risk of not immunizing a baby are greater than the dangers of side
effects
 Explain to her importance of following schedule

CMT BANK OF QUESTIONS Page 76


REVIEW QUESTIONS PAEDIATRIC

2. A 2 years old child has been brought to your dispensary with the history of fever, cough,
vomiting, inability to breastfeed and difficulty in breathing for the past 2 days. Mother also
reported that the child had convulsion few hours ago. On examination: Respiratory rate is 58
breaths/minute and child has nasal flaring and lower chest wall in drawing. Explain the
management of this child and complications which may arise.
Diagnosis
 Severe pneumonia

Differential diagnoses
 Severe malaria
 Meningitis
 Septicemia
 Pulmonary edema

Treatment plan
 Inj. Benzyl penicillin 200mg/kg stat.
 Inj. Chloramphenicol 100mg/kg stat.
 Tabs. Paracetamol 15mg/kg stat
 Inj. Diazepam 0.2 – 0.3 mls stat
 Refer the child urgently

Possible complications
 Lung abscess
 Atelectasis
 Pneumothorax
 Bronchiectasis
 Cardiac failure
 Meningitis
 Septicemia

3. You were called to review Jane in postnatal ward who is two (2) days old. She presented with
twitching, yellowish discolouration of the eye and inability to breastfeed. On examination you
find out Jane has temperature of 380C and lethargic. Jane’s mother had prolonged rupture of
membrane occurring more than 18 hours before delivery. Write an essay on the management
of the above patient and measures which can be taken to prevent such condition.
Diagnosis
 Neonatal sepsis/Sepsis
 This is a clinical syndrome of bacteraemia with systemic signs and symptoms of
infection occurring in the first 4 weeks of life (Neonatal period)

Differential diagnosis
 Meningitis
 Very severe pneumonia

CMT BANK OF QUESTIONS Page 77


REVIEW QUESTIONS PAEDIATRIC

 Birth asphyxia
 Respiratory distress syndrome (RDS)
 Congenital viral or protozoan diseases
 Pulmonary edema
 Congenital pulmonary anomaly

Investigation
 Blood culture and sensitivity
 Full blood picture (FBP)
 Erythrocyte sedimentation rate (ESR)

Treatment
 Broad spectrum antibiotics: Ampicillin 50ml/kg per dose plus Gentamycin 5mg/kg per
dose OR Cotrimoxazole 50mg/kg per dose.
 Give first dose and refer urgently to hospital

General supportive treatment


 Provision of warmth
 Respiratory and hemodynamic management

Prevention
 Good antenatal care
 Baby should be feed early and EBF
 Cord should be kept dry and clean
 Hand washing before and after touching the baby
 Nursery environment should be clean and dry
 All procedure in hospital should be performed with mask and gloves
 Avoid unnecessary invasive interventions
 Strict routine for washing, disinfection, cleaning of clots and incubation should be
ensured in health care setting.

4. Kandiga is a one (1) year old child. He was brought to your health center by his mother who
told you that kandiga was apparent well until 4 days ago when he started experiencing fever,
he was given paracetamol syrup and fever subsided. This morning fever recurred and he had
two episodes of convulsion that was tonic-clonic in nature. On examination, he is febrile with
temperature at 39.50C. MRDT is negative and RBG is 4mmol/l and FBP parameters were
within normal range. Explain the management of this child base on provisional diagnosis,
differential diagnosis, investigations and treatment of the above patient.
Diagnosis
 Febrile convulsion
 These are twitching that associated with significant rise in body temperature

Differential diagnosis

CMT BANK OF QUESTIONS Page 78


REVIEW QUESTIONS PAEDIATRIC

 Epilepsy
 Encephalitis
 Septicemia
 Meningitis
 Severe malaria

Investigations
 Blood culture
 Lumber puncture
 BS for MPS

Treatment
 Ensure airway is clear and child is breathing
 Position the child to avoid injury
 Stop convulsion by:
- Diazepam 0.5mg/kg per rectal (IV dose 0.2 -0.3 mg/kg)
- Hold buttocks together for few minutes
- Give up to 3 doses if convulsion continue after 10 to 20 minutes
- Avoid diazepam and use Phenobarbitone in infants < 2 week at age, 5-10mg/kg
IV every 15 to 30 minutes.
 Avoid oral medication until convulsion is controlled
 Give antipyretic
 Find and treat the cause

5. A 7 years old boy presents at your clinic with history of loss of consciousness for the past 3
hours. Prior to this, his parents report that the child has been losing weight despite of good
appetite in previous one month. On examination the child looks dehydrated, cachexic,
breathing rapidly with sweet odour. Discuss the diagnosis of this patient and its management
at health center level.
Diagnosis
 Diabetic ketoacidosis (DKA)
 This is a potentially life threatening complication in patients with diabetes mellitus. It
happens predominantly in those with type 1 diabetes, but it can occur in those with
type 2 diabetes under a certain circumstance.

Investigation
 Random blood glucose
 Urinalysis
 Blood smear
 Full blood picture to screen infections

Treatment

CMT BANK OF QUESTIONS Page 79


REVIEW QUESTIONS PAEDIATRIC

 Correct dehydration aggressively gives: 10-20ml/kg of NS or RL within 1st and 2nd


hour.
 Correct acidosis and reverse ketosis with insulin and bicarbonate. Give Insulin (short
acting) 0.3IU/kg stat. Avoid over hydration.
 Treat infections if identified
 Identify and treat precipitant events
 Put on NGT – if available
 Refer the patient to the hospital

Prevention
 Children with recurrent DKA should be evaluated for the cause
 Newly diagnosed diabetic patients should be followed up closely after hospital
discharge.
 Early and active involvement of the parent in the treatment of the child is essential to
prevent or delay relapse of ketoacidosis.
 Education to be given to patients should include
- Diet
- Insulin management
- Blood sugar monitoring
- Signs and symptoms of hyper and hypoglycaemia
- Psychological counseling to the child and the family
- Timely referral of children with symptoms of polydipsia, polyphagia and
polyuria.
- Promote breast feeding prevent or delay the occurrence of diabetes in children
- Early referral of any diabetic child with febrile illness

6. Describe the malaria in under-five children and its management at Hospital level
An introduction
 This is an illness characterized by nonspecific feature prior to the onset of sudden high
fever and chills.
 Uncomplicated malaria
- Fever is the most common feature
- Onset of symptoms may resemble flu-like illness.
- Child may present with cough, vomiting and diarrhea
- In infant: Refusal to breastfeed / poor appetite and restlessness
- Older children: Body malaise, headache, joint pains, body aches, poor appetite
and body weakness
- Pallor, hepatomegaly and enlarged spleen are more common in children < 5
years old
 Severe malaria
- Cerebral malaria
- Convulsion
- Impaired consciousness

CMT BANK OF QUESTIONS Page 80


REVIEW QUESTIONS PAEDIATRIC

- Change of behaviors; Hallucination, Delusion and Agitation


- Severe malaria anaemia with HB < 7g/dl or hematocrit < 21%
- Hyperparasitaemia > 5000 asexual parasities per 200 WBC (> 20,000 asexual
parasites per µl)
- Respiratory distress
- Hypoglycaemia, Blood glucose < 2.5 mmol/L
- Prostration/extreme weakness
- Circulatory collapse / shock
- Vomiting every thing
- Inability to breatfeed.
- Acute renal failure
- Jaundice
- Hemoglobinuria
- Bleeding tendency/Disseminated intravascular coagulation
Investigation
 Blood smear
 MRDT
 Random Blood Glucose
 FBP
 CXR in respiratory distress syndrome.

Treatment
 Uncomplicated
Neonate
- Parenteral ant-malaria drug
- Antipyretics PCM-Syrup (10-15mg) PO TDS for 3/7 days
- Give also broad spectrum antibiotics 1st dose (Ampicillin, Gent or
Chloramphenical)
- Give dextrose bolus if hypoglycemic then refer
- Give blood transfusion if Hb < 10g/dl (20mls/kg over 3-4 hrs)
3 months or weight above 5kg
- ALU
- Give antipyretic
- Ensure baby is feeding well.
 Severe
- Give artemether injection 4.2mg/kg (loading dose) then followed by 1.6mg/kg
IM daily for 6 days.
- Give bolus of Dextrose 10% 5mls IV stat if hypoglyceaemia
- Give – BT 20mls/kg IV
- Ensure baby is feeding well
Prevention/Control
 Prompt and effective treatment with Artemisin based combination therapy

CMT BANK OF QUESTIONS Page 81


REVIEW QUESTIONS PAEDIATRIC

 Use insecticides or mosquito repellant to avoid mosquito bite


 Eliminate mosquito breeding site e.g. Fill in water dishes
 Anti-malaria chemoprophylaxis to sicklers and non-immune travelers
 IPT – in pregnant women
 Use of ITNs
 Indoor residual spraying (IRS)

7. Anaemia is a manifestation commonly seen paediatric condition in many countries including


Tanzania. Discuss the management of iron deficiency anaemia
Introduction
 This is a low haemoglobin concentration in the blood below the reference range for
age and sex. It is caused by:-
- Inadequate intake of food rich in iron minerals
- Impaired absorption (Disorders of GIT or following gut surgery
- Increased loss (Acute chronic blood loss) e.g. from Hookworm infestation

Investigation
 Full blood count and blood film examination
 Stool analysis, iron panel and gastrointestinal endoscopy

General treatment
 Give diet rich in iron nutrients such as meat, vegetable
 Improve/change meal habit e.g. Drinking tea during meals. Tea interfere with
absorption.

Specific treatment
 Ferrous sulphate – 5mg/kg/day elemental Fe, in 1-2 divided doses orally for three
months
- Give fruits between meals for better absorption.
- Do not give in kwashiorkor or in severe infections in the acute phase
 Folic acid – 5mg/day all ages for 3 months in haemoglobinopathy: give lifelong
haemeolytic conditions – sickle cell and crisis anaemia after recovering from malaria
attack.
 Treatment of underlying causes e.g. Worms give Mebendaole or Albendazole

Prevention
 Promote breastfeeding and mixed complementary foods containing meat or fish, dark
green vegetables and fruits.
 Avoid tea during meals as it interferes with absorption
 Growth monitoring and immunization are essential
 Proper hygiene and sanitation
 Identify and treat helminthic infections
 Used ITNs to prevent malaria

CMT BANK OF QUESTIONS Page 82


REVIEW QUESTIONS PAEDIATRIC

 Give folic acid at the age of 4 weeks to 6 months and iron at age of 6 weeks to 6 months
to premature low birth weight infants and to twins.

8. An eight-year-old boy presents at health center with loss of consciousness for the past 1(one)
hour. Prior to this, parents report that the child has been losing weight despite of good appetite.
He was experiencing bed wetting. On examination the child is dehydrated, cachexic, febrile
T38.20C. there is fast breathing with sweet odour. From the presentation above, discuss the
management of this child
Diagnosis
 Diabetic ketoacidosis

Investigation
 Random blood glucose
 Urine for ketone bodies
 Blood smear for malaria parasites

Treatment
 Ensure clear airway and child is breathing
 IV Ringer lactate /NS
 IM Antimalarial drug
 IM Chloramphenical 2.5mg/kg stat
 Insert indwelling catheter
 Insert nasogastric tube
 Refer to the hospital

9. A four (4) year old girl presents at the health center with fever, vomiting, diarrhea and painful
micturition. On examination she is febrile with temperature of 38.50C, no signs of dehydration
and the child has normal breathing. Base on diagnosis and differential diagnoses, manage her
condition.
Diagnosis
 Urinary tract infection
 This is an infection of the urinary tract which comprises kidney, ureter and urinary
bladder

Differential diagnosis
 Malaria
 Acute watery diarrhea
 Septicemia

Investigations
 Urinalysis
 Urine for culture and sensitivity
 Ultrasound scan

CMT BANK OF QUESTIONS Page 83


REVIEW QUESTIONS PAEDIATRIC

Supportive treatment
 Ensure the child is taking enough amount of water
 Antipyretic; Give Paracetamol 10mg/kg tds for three days
 Encourage feeding

Specific treatment
 Parenteral antibiotics : Intramuscular Gentamicin 5mg/kg OD for 7 days

10. A 4 years’ old child with 12Kg sustained water splash burn injury on the anterior trunk and
both upper limbs while playing in the kitchen. He is brought to the health center 3 hours later.
Manage the above child and mention five (5) immediate complication that may get
Diagnosis
- Burn wound
Management
- ABC
- Fluid resuscitation
 Insert IV large bore cannula (16G or 14G) through unburned area of the skin
 Give Ringer’s Lactate by using Parkland formula = 4ml x % of BSA X body
weight (kg)
 BSA : Ant.trunk = 18
U.limbs = 18
Total BSA = 36
 Total fluid = 4ml x 36 x 12kg = 1728mls
 864mls is give in the 1st 8hours and another 864mls in the next 16hours
 3 hours has passed so in 1st 5hours remained = 540mls will be given.
 Then 864mls in the next 16hours.
- Catheterize the patient to monitor urine output. Urine output should be not less than
0.7ml/kg/hour. Electrolyte and fluid replacement will be guided by urine output.
- If the urine output is not adequate, increase infusion by 200ml net hour.
- Monitor vital signs
- Replacement should be assessed on the hourly basis
- In the 2nd 24 hours add 5% dextrose 150mls
- Social the surgical toilet of the wound
- Local antiseptics such as Silverex cream or Povidone iodine.
- Obtain blood for grouping and cross matching
- Dress fingers separately if involved in burns to avoid contractures.
Immediate complications
- Fluid and electrolyte imbalance – hypovolemic shock
- Acute renal failure - extensive burns
- Local wound sepsis – septicemia
- Anaemia
- Respiratory failure – smoke inhalation / CO intoxixation

CMT BANK OF QUESTIONS Page 84


REVIEW QUESTIONS PAEDIATRIC

11. A four (4) months old female baby was brought to the hospital with history of vomiting, fever
and convulsion. On examination she was febrile with temperature of 390C, rigid posture and
unequal pupil dilatation. Discuss the diagnosis and management of this patinet
Diagnosis
- Meningitis

Differential diagnosis
- Septicemia
- Severe malaria
- Tetanus
- Urinary tract infection
- Hypoglycemia

Other clinical feature


- Neck stiffness
- Positive Berning sign
- Positive Brudzinksi sign

Investigation
- Lumber puncture for CSF analysis – Appearance, pressure, protein, glucose, AFB,
culture and sensitivity and gram stain
- Blood slide for malaria parasites
- Urinalysis
- Random blood glucose/sugar
- Blood culture and sensitivity

12. A 2-year-old female child was brought to hospital with history of 1-week trauma of the left
lower leg i.e. she felt down and bruised her left mid lower leg. This was dressed at home and
was about to heal when suddenly she developed fever and a painful swelling on the site of
trauma:-
(a) What are the possible diagnoses?
- Acute osteomyelitis
- Cellulitis
- Abscess
(b) If acute osteomyelitis is the diagnosis what will be your management?
(c) What are the complication?
- Chronic osteomyelitis
- Leg length shortening
- Septicemia

13. Sikujua, a 4 years old child, is brought to the hospital by her parents because she has speech
ability of 1-year-old child, delayed use of gesture and unsocial behavior. Her mother had a

CMT BANK OF QUESTIONS Page 85


REVIEW QUESTIONS PAEDIATRIC

difficult labour, and suffered meningitis as result of otitis media with effusion at 3 month of
age. On examination Sikujua show significant intellectual functioning
(a) What are Sikujua’s problem?
- Speech delay
- Hearing loss – Sensoryneural
- Motor cerebral palsy
(b) What are the causes of those problems?
- Obstructed labour
- Meningitis
- Otitis media
(c) What is the treatment approach?
- Speech therapy
- Occupational therapy
- Physiotherapy
(d) Outline the care plan for Sikujua

14. A 3 years old girl was brought to the hospital with her mother with history of cough and
difficulty in breathing; her mother also says her child is unable to drink. One examination
there was lower chest in drawing. Explain the management of the above patient, the causative
organisms and complications which may occur.
Diagnosis
- Very severe pneumonia
- This is the infection of the lower respiratory tract characterized by inflammation of the
lung parenchyma with consolidation of alveoli.
Differential diagnosis
- Asthma
- Bronchiolitis
- Bronchitis
- Lung abscess
- Foreign body aspiration
- Severe malaria
Causative organism
- Streptococcus pneumonia
- Hemophilus influenza
- Group A streptococcus
- Mycoplasma pneumoniae
Investigation
- Chest X-ray
- Full blood picture (FBP)
- Blood slide of malaria parasite
Treatment
- Manage airway
- Ampicillin 200mg/kg IV in 4 divided dose plus Gentamicin 5mg/kg once per day
- Antipyretics e.g. Paracetamol 15mg/dose

CMT BANK OF QUESTIONS Page 86


REVIEW QUESTIONS PAEDIATRIC

- Oxygen – Use nasal prongs or nasal catheter or nasopharyngeal catheter.

15. A 8 years old boy was brought to the hospital by his mother with the history of fever, vomiting
everything and convulsion for one day. On examination he was alert, febrile with temperature
of 38.50C; pale, not jaundiced and body weight 25 Kgs. His Hb was 4.5g/dl. Explain the
management of this patient above and preventive measures that can be taken
Diagnosis
- Severe malaria
- This is a parasitic and chronic protozoan infection caused by plasmodium which infect
red blood cells.
Differential diagnosis
- Meningitis
- Septicemia
- Tetanus
- Urinary tract infection
- Encephalitis
Investigations
- Blood slide for malaria parasites
- Random blood glucose
- Full blood picture (FBP)
- Urinalysis
- Serum creatinine
- Liver function tests (bilirubin, AST, ALT, ALP)
- Serum electrolytes
- Blood culture and sensitivity
Treatment
- IV artesunate 60mg at 0, 12, 24 hours then Tabs ALU 3 tablets at 0, 8 hours then bid
for 2 days.
- Blood transfusion whole blood 1 unit then continue with tabs of ferrous sulphate
200mg bid for 1 months.
- Antipyretics e.g. Paracetamol 250mg tds for 3 days.
- Anticonvulsant if the child is convulsing i.e. Diazepam
Monitoring
- Fluid input and output chart
- Level of consciousness
- Temperature, PR, RR and BP
- Feeding
- Changing position every 2 hours
Prevention
- Prompt and proper treatment with Artemisia based combination therapy
- Mosquito bite avoidance especially during night using mosquito coils, mosquito
meshing, insect side treated nets.
- Vector control, indoor spraying, eliminate mosquito breeding sites.
- Drugs prophylaxis for sickler and visitor

CMT BANK OF QUESTIONS Page 87


REVIEW QUESTIONS PAEDIATRIC

16. Describe the management of a 4 days old female baby who presents with hematoma on the
skin following venipuncture during drawing of blood for laboratory investigation. On
examination, the child is alert and febrile. However umbilical stamp was still oozing blood.
In your essay include diagnosis, clinical signs you will look for, common causes of the above
diagnosis, investigations and treatment
Diagnosis
- Hemorrhagic disease of new born

Clinical signs
- Bleeding (spontaneous or at injection site)
- Pallor
- Sign of shock
- Prolonged bleeding time
- Prolonged prothrombin time

Common causes
- Deficiency of vitamin K- dependent clotting factors, factors II, VII, IX and X.

Investigations
- Hb estimation
- Blood grouping and X-matching
- Liver function test
- Bleeding time
- Prothrombin time

Treatment
- Stop visible bleeding with direct pressure and bandage
- Give first dose of antibiotics if there is sign of infection
- I.M vitamin K1 1-2mg stat
- Blood transfusion with fresh blood
- If at dispensary level refer to hospital

17. A 3-months-old child was brought to the hospital by his mother ho reports that the child has
feeding difficulties, inability to suck and has rigid jerk movements, mainly of proximal parts
of the arms and legs. Mother denies history of fever. On examination: child was hypotonic
and floppy. Describe the management of this child.

18. A 4-year-old boy was brought to the hospital with complaints of sudden onset of abdominal
pain, vomiting and diarrhea for one day. The boy had episodes of screaming and sweating.
Had no fever, vomited many times and also had watery stools many times. On further
questioning revealed initially stool was watery but currently passing bright red blood per
rectum. On examination; he was ill looking, dehydrated, had abdominal distension which was
tender and palpable mass which was sausage shaped extending from the right iliac region to
the umbilicus. Describe the management of this patient.

CMT BANK OF QUESTIONS Page 88


REVIEW QUESTIONS PAEDIATRIC

19. A 15-month old child brought to the dispensary with a main complain of high grade fever and
passing coca-cola like urine without discomfort on micturition for 2 days. In addition, a history
of one episode of convulsion was also reported. On examination: There is no features of
meningeal irritation; he was lethargic with mild palmar pallor and vital signs were Respiratory
rate: 26 breaths per minute, Pulse rate: 98 beats per minute and body temperature of 39.80C.
Describe the management of this patient.

20. A 4-year-old boy is brought to the dispensary with main complaints of acute onset abdominal
pain and passing loose like black current like stool. Clinical findings revealed a distended and
tender abdomen with sausage – shaped mass felt on the epigastrium. Describe the management
of this patient.

21. A 6-months-old child presents to hospital with difficulty in breathing for 2 weeks which is
progressively increasing in severity and is associated with non-productive cough. He has used
several antibiotic without relief. His mother test HIV positive during pregnancy and was
started on ART. There is no open TB contact, no excessive night sweat. On examination:
afebrile, dyspneic, oxygen saturation 89%. MUAC 12cm, weighs 7kgs, and has normal chest
examination. Describe the management of this child.

22. A 5-day-old baby presents at the hospital with fever, twitching and inability to breastfeed.
This morning he convulsed twice and became unconscious. On examination: Unconscious,
febrile (T=400C), bulging fontanelle, stiff neck, no pallor, no jaundice, respiratory rate
=64bpm and heart rate=164bpm and weighs 3.5kg. The rest physical examination is
unremarkable. Describe the management to this child and complications.

23. A 5-year-old child was brought to the health center with the complaints of difficulty in
breathing for 2 days. No history of cough or open TB contact. Since the onset of illness the
child has not been eating well due to difficulty in breathing. On examination: febrile, mild
pale, dyspneic, RR=42cycles/minute, chest wall indrawing and cyanotic with oxygen
saturation of 85%, clear chest of auscultation. Describe the management of this child.

24. A 10-year-old female was brought to the hospital with complaints of facial swelling for 4
weeks. The swelling is more during morning and subside during day time. Her mother
reported to note the decrease in output of urine. There is no history of cough, difficulty in
breathing or fever. On examination: Puffy face with lower limb pitting oedema. Other
examination findings are normal. Describe the management of this child and complications
which are more likely to occur.

25. A 8-year-old girl who is on ART for 3 years was brought to the hospital with main complaints
of dry cough and difficulty in breathing for 1 day. There is no history of convulsion, lower
limb swelling or night sweating. On examination: alert, dyspneic, no cyanosis noted.
Respiratory examination; reduced breathing sound otherwise no crackles and rhonchi. Vital

CMT BANK OF QUESTIONS Page 89


REVIEW QUESTIONS PAEDIATRIC

signs: T=38.50C, RR=40cpm, PR=110bpm and SPO2=86%. Describe the management of this
child.

26. A 4-year-old boy was brought to the health center with complaints of abdominal discomfort,
passing loose stool for 2 days. Her mother reports the stool to be water in nature. He had about
5 motions per day. There is no history of discomfort during micturition. On examination; the
child is lethargic with sunken eyes. Skin turgor goes back slowly. Vitals; T=37.30C,
PR=98bpm, RR=24bpm. His body weight is 15kg. Describe the management of this child.

27. A 10-year-old boy was brought to the hospital with complaints of fever 3 days and confusion
for the past 3 hours. No history of convulsion or loss of consciousness. Her mother report that
he got injured on his leg about 5 days ago. On examination: dyspneic, mild pale, conjunctiva
jaundice, extremities are cold with delayed capillary refill. Vitals; T=39.80C, PR=120bpm
with weak volume. Describe the management of this child.

28. A 3-month-old baby was admitted at the hospital with complaints of fever, dry cough= and
difficulty in breathing for three days. On examination; alert, grunting, dyspneic with nasal
flaring. On respiratory examination; bilateral crackles with bronchial breathing sound.
T=390C, RR 88bpm, PR=102bpm. Describe the management of this patient and complications
which may occur.

CMT BANK OF QUESTIONS Page 90


REVIEW QUESTIONS PAEDIATRIC

CMT BANK OF QUESTIONS Page 91


REVIEW QUESTIONS PAEDIATRIC

CMT BANK OF QUESTIONS Page 92


REVIEW QUESTIONS PAEDIATRIC

CMT BANK OF QUESTIONS Page 93

You might also like